Você está na página 1de 131

www.MRCPass.

com

Rheumatology



elzohryxp@yahoo.com

https://www.facebook.com/elzohryxp

May2012

www

w.MRCPass.ccom Rheum
matology

RheumatollogyQ0011

A355yearoldm manpresen ntswitha6


6monthhisstoryofarthhralgia,mouthulcerattionand
eye irritation.OOnexaminaation,hehaadsomeulccerationintthemouth, bilaterally swollen
wrisstsandredu ucedrangeofmovemeentsofboth hknees.
His investigatioonsshowed d:whitecelllcount11 x10^9/L,C
Creactivep
protein100 0mg/dl,
Rheeumatoidfactornegativve.

Wha atisthelikeelydiagnosiis?
A.Reiterssyndromee
B.Sjogre
enssyndrom
me
C.Ankylosingspondylitis
etssyndrom
D.Behce me
E.Sarcoidosis

Answer:d)Beh
hcetssyndrrome.

Behhcets syndrome is a multisystem m disorder characterissed by recu urrent oral genital


ulceeration,eyeelesions(an
nteriorand dposterior uveitisorrretinalvascu ulitis),skin lesions,
(eryythemanod dosum,papu ulopustularrlesionsfollliculitis)and
dapositivepathergyteest.
Thee pathergy phenomenon is considered an outstanding feature of o Behcet disease.
d
Following a neeedle prick or intradermal injection with saaline or dilu
ute histamiine, the
punncture site becomes inflamed and deveelops a sm mall sterilee pustule due to
hyperactivityooftheskinto
oanyintraccutaneousinsult.


OralUlceration
ninBehcet'sdisease

Dr.K
KhalidYusuffElzohrySo
ohagTeachin
ngHospital2012
2
www.MRCPass.com

Rheumatology

RheumatologyQ002

A 33 year old female with systemic lupus erythematosis has arthralgia involving her
upperlimbs.Shealsohasabutterflyfacialrashandarashonthetrunk.
UrinedipstickshowsnoProteinuriaorhaematuria.Herrenalfunctionisnormal.

Whichoneofthefollowingmedicationsismostappropriate?
A.Methotrexate
B.Prednisolone
C.Azathioprine
D.Hydroxychloroquine
E.Cyclosporin

Answer:D)Hydroxychloroquine.

NSAIDsandhydroxychloroquineareusedforskininvolvementandarthritis.
NSAIDs are used for mild disease. Hydroxychloroquine is useful for disease not
controlled by NSAIDS. Steroids are used in moderate to severe disease.
Immunosuppressive treatments such as azathioprine and cyclophosphamide are used
typicallywhenthereisrenalorcerebaldisease.

Dr.KhalidYusufElzohrySohagTeachingHospital2012
3
www.MRCPass.com

Rheumatology

RheumatologyQ003

A 75 year old woman with longstanding Rheumatoid arthritis has great difficulty
walkingandcommentsthatbothherlegsarestiffandjumpy.

Whatisthemostlikelycauseofthepresentation?
A.Ankylosingspondylitis
B.Syringomyelia
C.Osteoporosis
D.Atlantoaxialinstability
E.Disccompression

Answer:d)Atlantoaxialinstability.

CervicalcordcompressionduetoatlantoaxialinstabilityisthemostlikelycauseofUMN
nerveweakness.

Dr.KhalidYusufElzohrySohagTeachingHospital2012
4
www.MRCPass.com

Rheumatology

RheumatologyQ004

A50yearoldwomanhassevererheumatoidarthritis.Sheisadmittedwithworsening
breathlessness.
She is currently on ibuprofen, methotrexate, celecoxib, and paracetamol. On
examination,shehasfeaturesofrheumatoidchangesinherhandsandlookspale.There
isnopalpablelymphadenopathyandnoabdominalmasses.
InvestigationsshowedthatherHblevelis7.8g/dlwithaMCVof90,WCC2.1x10^9/l
andplatelets55x10^9/l.
Reticulocytecountis0.3%(0.5%to1.5%).

Whatisthelikelycauseoftheanaemia?
A.NSAIDuseandGIbleed
B.Treatmentwithcelecoxib
C.Treatmentwithmethotrexate
D.Anaemiaofchronicdisease
E.Felty'ssyndrome

Answer:c)Treatmentwithmethotrexate.

The patient has an aplastic anaemia which can be caused by methotrexate or


azathioprine,DMARDs(penicillamineorgold).
Abnormallylowreticulocytecountcanbeattributedtochemotherapy,aplasticanemia,
perniciousanemia,bonemarrowmalignanciesandlowerythropoietinlevels.

Dr.KhalidYusufElzohrySohagTeachingHospital2012
5
www

w.MRCPass.ccom Rheum
matology

RheumatollogyQ0055

A655yearoldw womanpressentswithd dryeyesandadrymouth.Herinvvestigationssshow:


ANA A strongly positive (11:1600), anttiRo/SSA an
ntibodies strongly
s positive, rheu
umatoid
facttorpositive,,IgGmarkedlyelevatedat42g/l
(norrmal<15g//l),IgMandIgAlevelsaarenormal.

Wha atisthelikeelydiagnosiis?
A.Mono
oclonalgamm
mopathyofu
unknownsiggnificance
B.PrimaarySjogren'ssSyndrome
C.SystemicLupusErrythematosu
us
D.Reiter'ssyndromee
E.PolyarteritisNodo
osa

Answer:b)Prim
marySjogreen'sSyndrom
me.

Theeclinicalfeaturesandaareconsisteentwithprim
marySjgreensSyndrom me.
Hyppergammaglobulinaem miaispresen ntin80%offindividualss.ANA,AntiRo/SSAanttibodies
are presentin approximattely90%offindividualssandthere canalsobe eaweakly positive
rheuumatoidfacctor.


DrymouthseeninSjogren
n'ssyndrom
me

Dr.K
KhalidYusuffElzohrySo
ohagTeachin
ngHospital2012
6
www

w.MRCPass.ccom Rheum
matology

RheumatollogyQ0066

A555yearoldffemalehas beenonlongtermsteeroidsforchronicobsttructivepulmonary
diseease.Sheco omplainsoffpaininherrrightgroin nradiatingddowntheanteromediaalthigh.
Shehasanantalgicgait.
On examinatio d range of movement especially flexion,
on of the hip, there iss decreased
abductionandinternalrottation.

Wha atisthelikeelydiagnosiis?
A.Osteo
oarthritis
B.Rheumatoidarthrritis
C.Metastatichiplession
D.Avasccularnecrosiisofthefem
moralhead
E.Hairlin
nefracture

Answer:d)avascularnecrosisoftheffemoralheaad.

In a
a patient on long term
m steroids presenting with groinn pains radiiating to th
he thigh
assoociatedwithhanantalgicgaitand decreased rangeofmovementoffthehip,th hemost
likelydiagnosissisavasculaarnecrosisofthefemooralhead.
Inth
hisconditio
on,MRIisth hemostsennsitiveand specifictecchniqueand disusefulfforearly
diaggnosis befo
ore collapsee of bone occurs. CT scan and xray are useful to rule
r out
advanceddiseaaseifdurationisnotclear.Bonesscanningismoresensiitivethanxraybut
isno
onspecific.

MRIshowingavascularneecrosisofth
hefemoralh
head

Dr.K
KhalidYusuffElzohrySo
ohagTeachin
ngHospital2012
7
www.MRCPass.com

Rheumatology

RheumatologyQ007

A60yearoldwomanontreatmentforlongstandingrheumatoidarthritispresentswith
breathlessness.Shecomplainsofadrycough.Theoxygensaturationwasfoundbe85%
onair.
Thechestxrayshowsadiffusebilateralinterstitialinfiltrate.
Bloodculturesandsputumculturesarenegative.

Whichdrugislikelytohavecausedthisadverseeffect?
A.Cyclosporin
B.Cyclophosphamide
C.Gold
D.Methotrexate
E.Sulphasalazine

Answer:d)methotrexate.

Pneumonitisisaseriousandunpredictablesideeffectoftreatmentwithmethotrexate
(MTX)thatmaybecomelifethreatening.Chestradiographyrevealsadiffuseinterstitial
ormixedinterstitialandalveolarinfiltrate,withapredilectionforthelowerlungfields.
Pulmonaryfunctiontestsshowarestrictivepatternwithdiminisheddiffusioncapacity.
Lung biopsy reveals cellular interstitial infiltrates, granulomas or a diffuse alveolar
damagepatternaccompaniedbyperivascularinflammation.

Dr.KhalidYusufElzohrySohagTeachingHospital2012
8
www

w.MRCPass.ccom Rheum
matology

RheumatollogyQ0088

A 65 year old man preseents has reccently deveeloped an acutely


a pain
nful right kn
nee. On
examination,h hehadatemmperatureo of37.2Cah hotandswo ollenrightkknee.
His whitecellccountwhich hwasraised dat14x100^9/l.AkneeeXraysho owedreducedjoint
spacceandchon ndrocalcinoosis.
Culttureofaspiratedfluidsshowednogrowth.

Wha atisthelikeelydiagnosiis?
A.Psoriaaticarthropaathy
B.Rheumatoidarthrritis
C.Pseud
dogout
D.Septicarthritis
E.Osteo
omalacia

Answer:c)pseudogout.

Theeclinicalpictureistypiccalofpseud
dogout.Calcificationo ofthearticu ularcartilage
ewould
be consistent and the cu ulture results excludes septic arrthritis. Thee diagnosis can be
confirmed byy the pressence of calcium pyrophosph
p hate crystaals in join nt fluid
demmonstratinggapositiveb birefringencceunderpo
olarisedlighht.


Calcciumpyroph
hosphatecrrystals

Dr.K
KhalidYusuffElzohrySo
ohagTeachin
ngHospital2012
9
www.MRCPass.com

Rheumatology

RheumatologyQ009

A60yearoldmanpresentswithapainfulswollenleftkneeaswellaspainandstiffness
ofbothhishands.
Onexaminationhisskinispigmented.Thereistendernessandswellingofthe2ndand
3rdmetacarpophalangealjointsofbothhands,hehashepatomegalyof8cmbelowthe
costalmargin.
Hiskneeisswollenandaspirationofthejointyieldsturbid,strawcolouredfluid.

Whichoneofthefollowingdiagnosisislikely?
A.Wilson'sdisease
B.Haemochromatosis
C.Pseudohypoparathyroidism
D.Pseudopseudohypoparathyroidism
E.Ankylosingspondylitis

Answer:b)haemochromatosis.

The history of abdominal pain together with skin pigmentation and hepatomegaly
suggest the patient has haemochromatosis. Haemochromatosis is associated with
chondrocalcinosis, which commonly affects the 2nd and 3rd metacarpophalangeal
joints.Theacutearthropathyislikelytobepyrophosphatecrystalarthritis.

Dr.KhalidYusufElzohrySohagTeachingHospital2012
10
www

w.MRCPass.ccom Rheum
matology

RheumatollogyQ0100

A700yearold manpresen ntswithbo


onyswellinggsoftheDIIPjointson
nbothhand
ds.They
werrepainfulayearagobuutarenowpainless.

Themostlikelyydiagnosisiis:
A.Heberdensnodes
B.Bouch
hardsnodess
C.Osler'snodes
D.Goutyytophi
E.Charccot'sjoints

Answer:a)Heb
berden'sno
odes.

Theemostlikelyydiagnosisiisosteoarth
hritis.AttheeDIPjoints,,swellingarreknownass
HebberdensnodesandattthePIPjoin nts,theyareeknownasBouchardssnodes.


Heb
berden'snode

Dr.K
KhalidYusuffElzohrySo
ohagTeachin
ngHospital2012
11
www.MRCPass.com

Rheumatology

RheumatologyQ011

A60yearoldmanpresentswithgoutytophi.Hehasbeencommencedonallopurinol
butdevelopsseverejointpains3dayslater.Onexamination,hehasatemperatureof
39Canderythematousswellingofhiswrists,kneesandankles.
Investigationsreveal:urate0.6(0.230.45),creactiveprotein180mg/L.

Whichofthefollowingislikelytohavecausedthepresentation?
A.Allopurinol
B.Colchicine
C.Prednisolone
D.Pseudogout
E.Septicarthritis

Answer:a)allopurinol.

Allopurinol blocks uric acid production and is the drug most often used in longterm
treatmentforolderpatientsandoverproducersofuricacid.Allopurinolistakenorally
once a day in doses of 100 mg to 600 mg, depending on the patient's response to
treatment.
Between 3% to 5% of patients experience leukopenia, thrombocytopenia, diarrhea,
headache,andfever.

Dr.KhalidYusufElzohrySohagTeachingHospital2012
12
www

w.MRCPass.ccom Rheum
matology

RheumatollogyQ0122

A 35 year old retired athlete presen


nts with sevvere burnin
ng pain affe
ecting the right
r leg
and foot. 4 months earlier, he had several arrthroscopic washouts for septic arthritis
a
affeectinghisrigghtknee,wwhichfollow
wedaninjuryy.
Examinationreevealsared ducedrangeofmovem mentofthee rightknee eandankle e. There
wassdiffuseswellingoftheerightlegaandfootwitthoverlyinggcool,scalyskin.

Wha atisthelikeelydiagnosiis?
A.Comp
partmentsyn
ndrome
B.Deepveinthromb
bosis
C.Reflexxsympatheticdystrophyy
D.Raynaaud'sphenomenonaffecctingtheleg
E.Venou
usvaricositiees

Answer:c)refleexsympath
heticdystrophy.

Thissclinicalsceenarioisco
ompatiblew withadiagn nosisofrefllexsympatheticdystro ophyor
commplex regio onal pain syndrome
s ffollowing the previou
us injury. Reflex
R symppathetic
dysttrophy, also o known ass RSD, is a condition of burning pain, stiffn
ness, swelling, and
disccolorationooftheaffecttedarea.
Theepainisoftensevereaanddisprop portionatettothesignssandfollow wsanonan natomic
distribution.Th heskinchanngesareduetotheasssociatedauttonomicdyysfunction.


Refllexsympath
heticdystro
ophyaffectingtherighttfoot

Dr.K
KhalidYusuffElzohrySo
ohagTeachin
ngHospital2012
13
www.MRCPass.com

Rheumatology

RheumatologyQ012

A 75 year old has had increasing back and leg pains several years. X rays reveal bony
sclerosis of sacroiliac, lower vertebral and upper tibial regions. He mentions greater
difficultyhearingovertherecentyears.
Bloodtestsrevealanelevatedserumalkalinephosphatase.

Whatisthelikelydiagnosis?
A.Paget'sdiseaseofbone
B.Osteoarthritis
C.Osteomalacia
D.Monoclonalgammapothyofuncertainsignificance
E.Multiplemyeloma

Answer:a)Paget'sdiseaseofbone.

In Paget's disease, onset of symptoms is usually insidious, with pain, stiffness, bone
deformity,headaches,decreasingauditoryacuity,andincreasingskullsize.
Signs may be bitemporal skull enlargement with frontal "bossing," dilated scalp veins,
nerve deafness in one or both ears, angioid streaks in the fundus of the eye, and
anterolateralbowingofthethighorlegwithwarmthandperiostealtenderness.
Pageticlesionsaremetabolicallyactiveandhighlyvascularandmayleadtohighoutput
heartfailure.
Deformitiesmaydevelopfrombowingofthelongbonesorosteoarthritisofadjacent
joints.
Pathologicfracturesmaybethepresentingfinding.Characteristicxrayfindingsinclude
increased bone density, abnormal architecture, cortical thickening, bow ing, and
overgrowth.
Biochemistry includes elevated serum alkaline phosphatase (or bonespecific alkaline
phosphatase)andincreasedurinaryexcretionofpyridinolinecrosslinks.Serumcalcium
andphosphoruslevelsusuallyarenormal,butserumcalciummayincreaseduringbed
rest.

Dr.KhalidYusufElzohrySohagTeachingHospital2012
14
www

w.MRCPass.ccom Rheum
matology

RheumatollogyQ0133

A 55 year old patient preesents with


h a tw o week
w historyy of pain and stiffnesss in her
shouldersand wrists.Theesymptomssimproveo overthedayy.Onexam mination,theerewas
synoovitisofbotthwristsan
ndproximalmusclewasting.HerEESRwas40mm/hr.

Wha atisthelikeelydiagnosiis?
A.Derm
matomyositis
B.Systemiclupuserrythematosus
C.Rheum
matoidarthrritis
myalgiarheumatica
D.Polym
E.Osteo
oarthritis

Answer:c)rheumatoidartthritis.

Inamiddleageedfemale, acutearthrritisofshou
uldersandw
wristsalonggwithsynovvitisare
high
hlysuggestiveofacuteRhematoid dArthritis.


Synovitisinvolvvingthewriistinrheum
matoidarthrritis

Dr.K
KhalidYusuffElzohrySo
ohagTeachin
ngHospital2012
15
www.MRCPass.com

Rheumatology

RheumatologyQ014

A 45 year old woman presents with numbness and weakness of her upper and lower
limbs.Shedevelopedasthmaattheageof30.
Onexaminationshelooksunwell.Shehaspalpablepurpuraoverherfaceandoverher
elbowsandknees.
Onneurologicalexaminationshehasarightsidedwristdropandthereisweaknessof
dorsiflexion of her right foot. Sensation is also impaired over the dorsum of her right
foot.
Investigations:
CXRisnormal.
Hb10.9g/dL
MCV90fl
WBC23x10^9/l
Eosinophils12%
ANCAnegative

Whatisthelikelydiagnosis?
A.Polyarteritisnodosa
B.ChurgStrausssyndrome
C.SLE
D.Allergicbronchopulmonaryaspergillosis
E.Takayasu'sarteritis

Answer:b)ChurgStrausssyndrome.

Thecombinationofmononeuritismultiplex,asthma,eosinophilia.
The American College of Rheumatology (ACR) has proposed 6 criteria for diagnosis of
ChurgStrausssyndrome.Thepresenceof4ormorecriteriayieldsasensitivityof85%
andaspecificityof99.7%.
Thesecriteriaare
(1)asthma(wheezing,expiratoryrhonchi)
(2)eosinophiliaofmorethan10%inperipheralblood
(3)paranasalsinusitis
(4)pulmonaryinfiltrates(maybetransient)
(5)histologicalproofofvasculitiswithextravasculareosinophils
(6)mononeuritismultiplexorpolyneuropathy

Dr.KhalidYusufElzohrySohagTeachingHospital2012
16
www

w.MRCPass.ccom Rheum
matology


Vasculiticrashontheskin
nofapatien
ntwithChurrgStrausssyyndrome

Dr.K
KhalidYusuffElzohrySo
ohagTeachin
ngHospital2012
17
www.MRCPass.com

Rheumatology

RheumatologyQ015

A40yearoldmanhasgeneralisedjointpainsandstiffness,particularlyintheknees.He
alsohassore,dryeyesanddifficultytoleratingcontactlenses.Onexaminationthereare
nojointswellingsoreffusions.HisESRis80mm/hour,RheumatoidFactorpositivewith
atitreof1/1024.

Whichofthefollowingislikely?
A.PositiveantibodiestoRoandLaantigens
B.PositiveantiSCL70antibody
C.Positiveantimitochondrialantibodies
D.PositiveantiSmantibodies
E.PositiveANCA

Answer:a)positiveantibodiestoRoandLaantigens.

RoisalsoknownasantissAandLaisknownasantissBantibody,botharediagnostic
tests for Sjogrens. The history of dry eyes (keratoconjunctivitis sicca) and joint pains
withstronglypositiveRhFgoeswithSjogrens.

Dr.KhalidYusufElzohrySohagTeachingHospital2012
18
www.MRCPass.com

Rheumatology

RheumatologyQ016

A45yearoldmanhasrecurrentsinusitisandhaemoptysis.Healsohashaematuriaand
mildrenalimpairment.
TestsaresentforsuspectedWegener'sgranulomatosis.

WhichoneofthefollowingtestshasgreatestspecificityforWegener'sgranulomatosis?
A.Antiglomerularbasementantibody
B.PANCApositiveantibodiesproteinase3
C.PANCApositiveantibodiesmyeloperoxidase
D.CANCApositiveantibodiesproteinase3
E.CANCApositiveantibodiesmyeloperoxidase

Answer:d)cANCApositiveantibodiesproteinase3.

Onimmunofluoresecnce,ifANCAarepresent,thestainingpatternmaybecytoplasmic
(cANCA)orperinuclear(pANCA).TypicalantigenspecificityincludescANCAproteinase
3 which is more common in Wegener's granulomatosis. p ANCA myeloperoxidase is
morecommoninpolyarteritisnodosa.

Dr.KhalidYusufElzohrySohagTeachingHospital2012
19
www.MRCPass.com

Rheumatology

RheumatologyQ017

Aladywithwhiplashinjury5yearsagopresentswithpainsintheneckandshoulder.
Theywerenotrelievedby12cocodamolsaday.

Whatshouldbedonenext?
A.Amitryptilline
B.NSAIDs
C.Physiotherapy
D.Morphine
E.Gabapentin

Answer:C)physiotherapy.

Whiplash injuries and radiculopathies causing back pains can be difficult to treat with
medications. Physiotherapy has an important role for symptom relief in combination
withanalgesia.

Dr.KhalidYusufElzohrySohagTeachingHospital2012
20
www.MRCPass.com

Rheumatology

RheumatologyQ018

A 45 year old man presents with a week history of a painful right leg. He has had
previous episodes of erythema nodosum, recurrent oral and scrotal ulceration.
Examinationrevealsadiffuselyswollenleftleg.

Whatisthelikelycauseofhisswollenleg?
A.Baker'scyst
B.Gonococcalarthritis
C.Reactivearthritis
D.Deepveinthrombosis
E.Cellulitis

Answer:d)Deepveinthrombosis.

TheoveralldiagnosisisBehcet'ssyndrome.Thereisathrombotictendency,hencethe
likelycauseofaDVT.

Dr.KhalidYusufElzohrySohagTeachingHospital2012
21
www

w.MRCPass.ccom Rheum
matology

RheumatollogyQ0199

A455yearoldwwomanpressentswithp paininthewrist.Thep
painiscenttredoverth
heradial
stylo ncreasedbyyabductionofthethum
oidandisin mbagainstresistance.

Wha atisthemoostlikelydia
agnosis?
A.Carpaaltunnelsyndrome
B.Rheumatoidarthrritis
C.DeQu
uervainsten
nosynovitis
D.Osteo
oarthritis
E.Adducctorpollicisssynovitis

Answer:c)DeQ
Quervain'stenosynovittis.

TheepainintheethumbonresistedabductionisttypicalofDe eQuervains.
De Quervain's disease occurs
o moree frequenttly in w om men. The age
a group usually
affeectedis30tto50yearo olds.
Thee history often is of unaccustomeed or excessive activity such as rose pruning. The
patiientcomplaainsofpainontheradiialsideofth hewrist.Ab bductionofthethumbagainst
resistance is painful.
p Finkkelstein's teest is posittive. This iss performed with the thumb
flexedacrossth hepalmofthehand,aaskingthep patienttom movethewrristintoflexxionand
ulnaardeviation n.
Thiss stresses the
t tendonss of abducttor pollicis longus and d extensor pollicis breevis and
reprroducestheepainofdeeQuervain'sstenosynovvitis.


Finkkelstein'steest

Dr.K
KhalidYusuffElzohrySo
ohagTeachin
ngHospital2012
22
www.MRCPass.com

Rheumatology

RheumatologyQ020

A43yearrheumatoidarthritiswasinvestigatedroutinelyandhadthefollowingresults:
Haemoglobin11.2g/dL
Platelets385x10^9/L
WhiteCellCount8.2x10^9/L
MCV110fL

Whichdrugisshelikelytohavebeenon?
A.Aspirin
B.Rituximab
C.Methotrexate
D.Vincristine
E.Hydroxychloroquine

Answer:C)Methotrexate

Methotrexate is associated with bone marrow suppression, and can lead to


pancytopeniaoramegaloblasticanaemia,especiallyiffolatetreatmentisnotgivenas
well.

Dr.KhalidYusufElzohrySohagTeachingHospital2012
23
www.MRCPass.com

Rheumatology

RheumatologyQ021

A75yearoldwomanhasrecentlybeencommencedonalendronateforosteoporosis.

Whatisthemechanismofactionofalendronate?
A.Increasesosteoblastactivity
B.Inhibitsosteoclastactivity
C.IncreasesvitaminDabsorption
D.Causeshypercalcaemia
E.Increasestheactionofoestrogenonbone

Answer:b)Inhibitsosteoclastactivity.

Alendronate is a bisphosphonate which can increase bone mineralisation by inhibiting


osteoclasticactivity.

Dr.KhalidYusufElzohrySohagTeachingHospital2012
24
www.MRCPass.com

Rheumatology

RheumatologyQ022

An 75 year old man presents with bilaterally painful knees. He has bilateral reduced
knee movements and crepitus. X ray shows sclerosis, osteophytes and loss of joint
space.

Whichoneofthefollowingisthemostappropriateinitialtreatment?
A.Ibuprofen
B.Ibuprofenandlansoprazole
C.Paracetamol
D.Codeinephosphate
E.Celecoxib

Answer:c)paracetamol.

Theprinciplegoalofsystemictherapyinosteoarthritisistoprovideeffectivepainrelief
with least associated toxicity. Paracetamol is the recommended initial therapy,
especiallyintheelderlyduetopossiblegastrointestinalupset.

Dr.KhalidYusufElzohrySohagTeachingHospital2012
25
www.MRCPass.com

Rheumatology

RheumatologyQ023

A35yearoldwomanrecentlyarrivedintheUKfromGhana.Shecomplainsofneckpain
withpinsandneedlesaffectingtherightarmassociatedwithaweakgrip.
Examination reveals tenderness over the cervical spine. X rays of the cervical spine
shownarrowingoftheC3/4andC4/5jointspaceandpartialcollapseofC4.
Investigationsshow:
Hb9.5g/dl
WCC11.1
platlets520
ESR120mminthefirsthour
CRP250g/l
Calcium2.21micromol/l
Albumin32g/l
alkalinephosphatase210units/l
phosphate0.8micromol/l.

Whatisthemostlikelydiagnosis?
A.Pottsdisease
B.Osteoporosis
C.Multiplemyeloma
D.Ankylosingspondylitis
E.Syringomyelia

Answer:a)Pottsdisease.

Potts disease is tuberculous infection of the spine with associated collapse of the
vetebral body. The infection spreads from tw o adjacent vertebrae into the adjoining
discspace.Ifonlyonevertebraisaffected,thediscisnormal,butiftwoareinvolved
theintervertebraldisc,whichisavascular,collapses.
Signs and symptoms include: Localised back pain, Paravertebral swelling, Neurological
signsincludingparaplegia.
Drug treatment (antituberculous drugs) is generally sufficient for Potts disease, with
spinal immobilisation if required. Surgery is required if there is spinal deformity or
neurologicalsignsofspinalcordcompression.

Dr.KhalidYusufElzohrySohagTeachingHospital2012
26
www

w.MRCPass.ccom Rheum
matology


Potttsdiseaseo
onanMRI

Dr.K
KhalidYusuffElzohrySo
ohagTeachin
ngHospital2012
27
www

w.MRCPass.ccom Rheum
matology

RheumatollogyQ0244

A 35
3 year old woman presents w with tight skin over her hand
ds with Raynauds
pheenomenon. Shehasulccerationof thefingerttipsandasssociatedsm mallwhitedeposits.
Shehasnotedincreasingb breathlessn
nessoverth
hepastfew years.Renaalfunctioniismildly
imppaired.

Wha atisthelikeelydiagnosiis?
A.SLE
B.Ataxiaatelangiectaasia
C.Polym
myositis
D.Limiteedcutaneou
usscleroderm
ma
E.Diffussecutaneoussscleroderm
ma

Answer:d)Lim
mitedcutaneeoussclerod
derma.

Limited cutan neous scleroderma isi also kn nown as CREST


C synddrome (calcinosis,
Rayynauds,(o)eesophagealdysfunction,sclerodacctyly,andteelangiectasia).Limiteddrefers
to the
t extent of skin invvolvement limited to the forearms and facce. They ge
enerally
devvelop pulmo onary hypeertension rather
r than
n pulmonarry fibrosis, leading to
ow ards
breaathlessnesss.


Tigh
htthickened
dskin(sclerrodactyly)seeninSclerroderma

Dr.K
KhalidYusuffElzohrySo
ohagTeachin
ngHospital2012
28
www

w.MRCPass.ccom Rheum
matology

RheumatollogyQ0255

A 70
7 year old d woman complains of o pain at the base of
o her right thumb. There
T is
tenddernessand ofrightfirstcarpometaacarpaljointt.
dswellingo

Wha atisthelikeelydiagnosiis?
A.DeQu
uervain'sten
nosynovitis
B.Rheumatoidnodu
ule
C.Osteo
oarthritis
D.Psoriaaticarthropaathy
E.Reiterr'ssyndromee

Answer:c)osteeoarthritis.

Osteeoarthritiso
ofthe1stccarpometacarpaljointiiscommon..Swellingissusuallybo
onyhard
dueeosteophyteformation n.PIPjointnodesarekknownasBo
ouchard'saandDIPjoinntnodes
areknownasH Heberden'snodes.

Dr.K
KhalidYusuffElzohrySo
ohagTeachin
ngHospital2012
29
www.MRCPass.com

Rheumatology

RheumatologyQ026

A 70 year old patient has a set of investigations due to lethargy. She complains of
polyuriaandgeneralisedbackpains.Uponinvestigation,shehasthefollowingresults:
Hb8g/dl
MCV100fl
Platelets190x10^9/l
totalprotein90g/l(6076)gm%
Albumin35(3749)g/l
calcium2.9(2.252.7)mmol/l
phosphate0.75(0.88)pmol/l

Whatistheappropriatenextinvestigation?
A.Completeliverfunctiontests
B.Urinaryalbumin
C.Plasmaelectrophoresis
D.24hoururineprotein
E.Uricacid

Answer:C)plasmaelectrophoresis.

The patient has multiple myeloma as indicated by hypercalcaemia, polyuria and bone
pains.

Dr.KhalidYusufElzohrySohagTeachingHospital2012
30
www.MRCPass.com

Rheumatology

RheumatologyQ027

A72yearoldladyhasseverepainintheleftkneeandrighthand.AnXrayofthehand
shows osteophytes and an X ray of the knee was normal. On examination, she has
limitedlefthipflexion.

Whatshouldbethenextinvestigationfortheknee?
A.MRIoftheknee
B.Bonescan
C.Arthroscopyoftheknee
D.CTscanoftheknee
E.Ultrasoundoftheknee

Answer:A)MRIoftheknee.

The patients presentation suggests osteoarthritis, but an MRI would be helpful to


exclude other pathology as well as confirm osteoarthritis in the context of a normal
kneeXray.TheMRIisusefulforassessingthestateofthecruciateligamentsandthe
joint cartilage as well as avascular necrosis. Arthroscopy is helpful but invasive in this
instance.

Dr.KhalidYusufElzohrySohagTeachingHospital2012
31
www.MRCPass.com

Rheumatology

RheumatologyQ028

A28yearoldpatientwithsystemiclupuserythematosusattendstheobstetricclinicat
25 weeks into her pregnancy. The foetal heart rate is 45 beats per minute. Foetal
echocardiographyshowscompleteheartblock.

Whichoneofthefollowingmaternalautoantibodiesislikelytobepresent?
A.AntiRo(SSA)
B.AntidsDNA
C.AntiLa(SSB)
D.AntiJo1
E.Anticentromere

Answer:a)AntiRo(SSA).

AntiRo antibody is associated with congenital complete heart block. When congenital
complete heart block occurs, SSA antibodies are almost alw ays present in maternal
andfetalserum(maternalantiRo(SSA)antibodycrossestheplacenta).

Dr.KhalidYusufElzohrySohagTeachingHospital2012
32
www.MRCPass.com

Rheumatology

RheumatologyQ029

A 34 year old lady with systemic sclerosis complained of lethargy. Her blood pressure
was185/90mmHg.
Fundoscopyshowedcottonwoolspots.InvestigationsshowedthatherU+Eswere:
sodium135mmol/l
potassium4.5mmol/l
urea12mmol/l
creatinine225mol/l

Whatisthetreatmentofchoiceforthispatient?
A.OralCaptopril
B.IVProstacyclin
C.IVLabetalol
D.IVSodiumnitroprusside
E.OralAtenolol

Answer:A)oralcaptopril.

Amajorcomplicationofsclerodermaisrenalcrisis.Thisischaracterisedbyabruptonset
ofseverehypertension,usuallyretinopathy,togetherwithrapiddeteriorationofrenal
functionandheartfailure.
Hypertension should be treated with an ACE inhibitor. This is because the underlying
pathology causing hypertension is angiotensin IIinduced vasoconstriction, and trials
have shown ACE inhibitors to have the best antihypertensive efficacy and improved
survival.
Theaimistoreducepressuregradually,asanabruptfallcanleadtocerebralischemia
/infarctions (as in any accelerated hypertension), and may cause decreased renal
perfusionaswellasacutetubularnecrosis.Calciumchannelblockersmaybeaddedto
ACEinhibitors.Parenteralantihypertensiveagents(suchasintravenousnitroprussideor
labetalol)shouldbeavoidedastheycauseabruptbloodpressuredrops.

Dr.KhalidYusufElzohrySohagTeachingHospital2012
33
www

w.MRCPass.ccom Rheum
matology

RheumatollogyQ0300

A 45 year wom
man presen nts with an 8 month history
h of jo
oint pains and stiffnesss of the
hanndsandfeett.
Examination reeveals a syynovitis of tthe distal interphalan
i ngeal joints,, left indexx finger,
righ
htwristandanklejoints.
Naillpittingwassnoticed.H
HerESRwass20mm/hr.

Whiichoneoffo
followingco onditionsisaassociatedwiththepa atternofjoinntinvolvem ment?
A.SLE
B.Psoriaasis
C.Rheum
matoidarthrritis
D.Septicarthritis
E.Reacttivearthritis

Answer:b)pso
oriasis.

Thee diagnosis is psoriattic arthritiss. Psoriatic arthritis is


i subclassiified according to
diffeerent patte
erns of arrthritis: asyymmetrical oligoarthritis, symmetric polyaarthritis,
spondyloarthro opathyand arthritismutilans.Inaabout20% ofpatientsthereisacchronic,
proggressive, deforming
d arthropath hy in an asymmetric
a cal pattern
n, includingg distal
inteerphalangeaaljointinvolvement.


PsoriaticArthro
opathyno
oteonycholyysisonthenails

Dr.K
KhalidYusuffElzohrySo
ohagTeachin
ngHospital2012
34
www

w.MRCPass.ccom Rheum
matology

RheumatollogyQ0311

A 16 year old girl presents with wid


despread paalpable purrpura over legs and buttocks.
Shehasabdom minalpainsaandisnoted dtohaveblloodandprroteinonurrinedipstickk.

Wha atisthemoostlikelydia
agnosis?
A.Juven
niledermatomyositis
B.Juven
nilechronicaarthritis
C.HenochSchnlein
npurpura
D.Rheumatoidarthritis
E.HUSTTTP

Answer:c)Hen
nochSchnlleinpurpuraa.

HennochSchnleinpurpurra(HSP)is asystemic vasculitism mostlyseen ninchildren n.Itisa


mulltisystemdisorderinvo olvingtheskkin,joints,ggastrointesttinalandren naltracts.
Aetiologyisunknown,buttthesyndro omeisoften nprecededbyinfectionssuchasG GroupA
betaa hemolyttic streptococcal resspiratory tractt infecttion, Campylobacter jejuni,
Myccoplasma pneumoniae
p e and virusees such as varicella, hepatitis
h B, EpsteinBarr virus,
andparvovirussB19.
Pathhologyisdu
uetointravasculardep positionofIgAimmuneecomplexesswithactivationof
com
mplement and
a leucocyyte infiltrattion. Patien
nts often present
p witth a purpuric rash
usually involving the butttocks and lower limb bs, arthralggia and join
nt swelling, severe
coliccky abdominal pain an
nd tenderness caused by vasculittisinduced thrombosis in the
gut..
Rennal involvem
ment comm monly preseents as miccroscopic haaematuria and proteinaemia.
Theemostserioouslongterm


com
mplicationfrromHSPisprogressiveerenalfailure.

Dr.K
KhalidYusuffElzohrySo
ohagTeachin
ngHospital2012
35
www

w.MRCPass.ccom Rheum
matology

RheumatollogyQ0322

A45
5yearoldm manhasdiffficultyben
ndingtouchinghistoesswhenben ndingover..Hehas
lowerbackpain,stiffness inthethorracicregionandreduceedchestexxpansion.Hislower
ne Xray iss shown ab
spin bove. He has
h negativve rheumaatoid factorr, and a HLAB27
H
genotype.
Wha atisthedia
agnosis?
A.Pagett'sdisease
B.Osteo
oarthritis
C.Marblebonedisease
D.Marfaan'ssyndrom
me
E.Ankylosingspondylitis

Answer:e)ankkylosingspo
ondylitis.

Ankkylosingspo ondylitisis aseronegativearthrop pathyassocciatedwith HLAB27ge enotype.


It leeads to inccreased stifffness and fusion of the
t vertebrrae causingg a bamboo o spine.
Typically,thesacroiliacjoiintsandhip pjointsareaffected,bu utthoracicandcervicaalspines
areeventuallyaffectedasswell.
Theere is a list of associattions mostlly starting with
w A. Theese are antterior uveittis/iritis,
aortticregurgitaation,aorticaneurysm m,apicalfibrosis,asperrgillosisinfeection,amyyloidosis
andarchilleste endinitis.
Inbbamboospine,thereissfusionoftthevertebrralbodiesandsquaringgoftheverrtebrae.
Onee also expeects syndesm mophytes (thin
( vertical dense sp
picules brid
dging the ve
ertebral
bod dies), Romano lesions (erosion att the cornerr of vertebral bodies) and enthessopathy
(ligaamentorteendoncalciffication).


Bam
mbooSpine

Dr.K
KhalidYusuffElzohrySo
ohagTeachin
ngHospital2012
36
www

w.MRCPass.ccom Rheum
matology

RheumatollogyQ0333

A35
5yearoldw womanpreesentswith redscalyp
plaqueson hercheeks andherfoorehead.
Onccloserexammination,therewasplu omehairfolliclesandatrophyofth
uggingofso heskin.

Whaatisthelikeelydiagnosiis?
A.Druginducedlupus
B.Psoriaasis
C.Discoidlupus
D.Sarco
oidosis
E.Lupusspernio

Answer:c)disccoidlupus.

Thee diagnosis is discoid lupus eryythematosu us. Lesions are discreete plaquess, often
erytthematous, scaly, with h extensionn into hair follicles.
f Th
hese lesionss can occurr on the
facee, scalp, in the pinnaee, behind tthe ears or on the neeck. There can also bee active
induuratederyth hemaandccentralatrophicscarrin ng.


Disccoidlupus

Dr.K
KhalidYusuffElzohrySo
ohagTeachin
ngHospital2012
37
www.MRCPass.com

Rheumatology

RheumatologyQ034

A 50 year old patient with active rheumatoid arthritis has failed treatment with gold,
methotrexate,hydroxychloroquineandsulphasalasine.Shehasongoingjointpainsand
erosivedamage.

Whatisthemostappropriatetreatment?
A.Highdoseprednisolone
B.COX2inhibitors
C.Infliximab
D.Ciclosporin
E.Azathioprine

Answer:c)infliximab.

The AntiTNF drugs are infliximab and adalimumab. The criteria for treatment with
these drugs are that patients who have been treated with at least tw o DMARDs
(diseasemodifyingdrugs)continuetohaveactiverheumatoidarthritis.
Examples of the DMARDs are: gold injections, sulphasalazine, hydroxychloroquine,
leflunomide, cyclosporin, azathioprine and methotrexate [One of the tw o must be
methotrexate].

Dr.KhalidYusufElzohrySohagTeachingHospital2012
38
www.MRCPass.com

Rheumatology

RheumatologyQ035

A 45 year old woman presents with a year's history of Raynauds phenomenon,


dyspepsiaandjointpains.Onexamination,shehassclerodactylyandsynovitisofsmall
jointsofherhands.HerESRis60mm/hr(<10)butRheumatoidfactorandAntinuclear
Antibodyarebothnegative.

Whatotherclinicalfeatureislikelyinthispatient?
A.Splinterhaemorrhages
B.Erythemamarginatum
C.Butterflyrash
D.Anterioruveitis
E.Smallbowelhypomotility

Answer:e)smallbowelhypomotility.

This woman features of a mixed connective tissue disorder such as CREST/systemic


sclerosis, sclerodactyly, Raynauds, dyspepsia and arthralgia. The other likely
developmentwouldbemalabsorptionwhichisassociatedwithhypomotilityofthesmall
bowel.

Dr.KhalidYusufElzohrySohagTeachingHospital2012
39
www.MRCPass.com

Rheumatology

RheumatologyQ036

A 60 year old man is on treatment for chronic heart failure with diuretics. He has
preivous history of athma. He presents with sudden onset of pain and swelling of the
metatarsophalangeal joint of his right big toe. Aspiration of the joint demonstrates
crystalsofmonosodiumurate.

Whatistherecommendedtreatment?
A.Aspirin
B.Colchicine
C.Nonsteroidalantiinflammatorydrugs
D.Cyclooxygenase2inhibitor
E.Allopurinol

Answer:b)Colchicine.

Inthisparticularpatient,colchicineisthebestoption.Inacutegout,eithercolchicineor
NSAIDscanbeused.However,asthmacontraindicatesNSAIDs.

Dr.KhalidYusufElzohrySohagTeachingHospital2012
40
www

w.MRCPass.ccom Rheum
matology

RheumatollogyQ0377

A35
5yearoldm manisreno ovatinghis apartment whenhesslamsadoo oragainsth
hisfoot.
He developssw welling,eryythemaand dpaininalllthedigits ofhisfoot.HehasannESRof
mm/houran
20m ndatemperratureof36 6C.

Wha atisthelikeelydiagnosiis?
A.Porph
hyria
B.Cellullitis
C.Gout
D.Raynaaud'sphenomenon
E.Reflexxsympatheticdystrophyy

Answer:e)reflexsympath
heticdystrophy.

Refllexsympattheticdystrrophyoccurrsfollowinggtraumato oaninjured dpartofth


hebody,
and can progrress to other parts. Itt is due to autonomicc nervous system
s dyssfuction.
Sym
mptomsofeextremepainandburningcanoccur.Analgesicsareoftenunhelpful.

Flusshing,shinyandatroph
hiedskinon
ntheleftleggindicatingreflexsymp
patheticdystrophy

Dr.K
KhalidYusuffElzohrySo
ohagTeachin
ngHospital2012
41
www.MRCPass.com

Rheumatology

RheumatologyQ038

A 42 year old lady has tightening of the skin around her hands and mouth. She has
severaltelangiectasiaonherhands,andcomplainsofseverecoldhandsinwinter.She
wasnoticedtobepale.Investigationsshow:
Hb4.5g/dl
MCV105fl
WCC6x10^9/l
platelets230x10^9/l
Iron22(1429)mol/l
Ferritin155(15200)mol/l
totalironbindingcapacity50(4572)umol/l
Folate11(320)g/l
VitaminB1285(120700pmol/l)

Whatisthelikelycauseofanaemia?
A.Perniciousanaemia
B.Secondaryfolatedeficiency
C.Celiacdisease
D.Methotrexate
E.Bacterialovergrowth

Answer:E)bacterialovergrowth.

Sclerodermacancausefolatedeficiencyduetomalabsorption.However,inthiscase,
thefolatelevelsarenormalandthereisB12deficiency.
B12deficiencycanoccurinconditionswherethereisbacteriaovergrowthinthesmall
intestine (blind loop syndromes) such as jejunal diverticulosis, Crohns disease, fistulas
and scleroderma. The anaerobic organisms metabolise vitamin B12 and impair
absorption.Whensystemicsclerosis(SSc)involvesthesmallintestine,normalperistaltic
movementsarelostandmotilityisimpairedleadingtostasisanddilatation.

Dr.KhalidYusufElzohrySohagTeachingHospital2012
42
www

w.MRCPass.ccom Rheum
matology

RheumatollogyQ0399

A555yearoldm
mandevelop pssuddeno onsetseverepaininhissrightbigtoe.
Onexaminationhehassw wellingofth hemetacarp pophalangeealjointofh hisrighthallux.The
surrroundskiniiserythemaatous.Itisttendertoto
ouch.Initialinvestigationsreveal araised
whitecell,counntandanelevatedCRP P.

Wha atshouldbeedonetoco
onfirmthed diagnosis?
A.MRIo
ofthetoe
B.Serum
muricacid
C.Serum
mrheumatoid
D.Jointfluidaspirateformicroscopy
E.Serum
mpyrophosp
phatelevels

Answer:d)join
ntfluidaspirateformiccroscopy.

Theelikelydiagnosisisacuutegout.Th heserumin nflammatoryymarkersaareraised, buturic


acid
dlevelsmayybenormall.
Join
nt fluid asp
pirate best test. Polarrised light microscopyy shows strrongly bireffringent
(neggativesign)needleshaapedcrystals.


AcuteGoutinvvolvingthelleftfoot

Dr.K
KhalidYusuffElzohrySo
ohagTeachin
ngHospital2012
43
www.MRCPass.com

Rheumatology

RheumatologyQ040

A 80 year old man developed acute monoarthritis of his right ankle following an
admissionwithcongestivecardiacfailure.Hementionedthathehadankleoedemaand
hadrecentlybeenprescribedfrusemidebytheGP.

Whatisthelikelydiagnosis?
A.Rheumatoidarthritis
B.Gout
C.Pseudogout
D.Osteoarthritis
E.Septicarthritis

Answer:b)Gout.

Goutcanbeprecipitatedbydiuretics,e.g.frusemide.

Dr.KhalidYusufElzohrySohagTeachingHospital2012
44
www.MRCPass.com

Rheumatology

RheumatologyQ041

A 45 year old has a past history of systemic sclerosis. She now has headaches and
blurredvision.Onexamination,shehasabloodpressureof220/100mmHgandthereis
evidenceofbilateralpapilloedema.

Whichofthefollowingmedicationsshouldbeprescribed?
A.Oralhydrochlorothiazide
B.OralLisinopril
C.SublingualNimodipine
D.IVSodiumNitroprusside
E.IVLabetolol

Answer:b)OralLisinopril.

A major complication of scleroderma is renal crisis which is characterised by abrupt


onsetofseverehypertension.Thehypertensionalmostalwaysisseverewithadiastolic
BPover100mmHgin90%ofpatients.Thereisassociatedhypertensiveretinopathyin
about 85% of patients. ACE inhibitors are first line, with an aim to reduce the blood
pressuregradually.

Dr.KhalidYusufElzohrySohagTeachingHospital2012
45
www.MRCPass.com

Rheumatology

RheumatologyQ042

A 50 year old man presents an acute monoarthritis of left knee. Gout is confirmed
followingjointaspirationandexaminationoffluidunderpolarisedlightmicroscopy.He
hadalsounderwentendoscopy3weeksearlierbecauseofindigestionthisconfirmeda
duodenalulcer.

Whichoneofthefollowingisthebesttreatmentforthepatient?
A.Allopurinol
B.Intraarticularcorticosteroidinjection
C.IndomethacinandLansoprazole
D.CelecoxibandLansoprazole
E.IndomethacinandMisoprostol

Answer:b)Intraarticularcorticosteroidinjection.

All nonsteroidals including Cox II selective inhibitors are relatively contraindicated in


the presence of active ulceration. In a large joint such as the knee, the safest option
wouldbeinjectcorticosteroidintothejoint.

Dr.KhalidYusufElzohrySohagTeachingHospital2012
46
www.MRCPass.com

Rheumatology

RheumatologyQ043

A 62 year old man has a 5 week history of pain and swelling affecting left knee, both
anklesandhisrightwrist.
Hehaslost6kginweight.
Hisinvestigationsshow:
WCC14.1X10^9/l
Hb10.3X10^9/l
MCV72fl
plt510X10^9/l
ESR63mminthefirsthour
CRP21g/l
CK120iu
RhF1/80
ANAnegative
ENAnegative
XRofhandsandfeetnormal.

Whatisthemostlikelydiagnosis?
A.Paraneoplasticsyndrome
B.Osteoarthritis
C.Polymyositis
D.Scleroderma
E.Behcet'ssyndrome

Answer:a)Paraneoplasticsyndrome.

The CK is not significantly raised. Weight loss and anaemia suggest underlying
malignancy.Paraneoplasticsyndromecanpresentwithanassymetricalarthralgiawhich
morecommonlyaffectsthelowerlimbs.Falsepositiverheumatoidfactorcanoccurbut
shouldbeoflowtitre.

Dr.KhalidYusufElzohrySohagTeachingHospital2012
47
www

w.MRCPass.ccom Rheum
matology

RheumatollogyQ0444

A 45 year old lady has h


had long staanding arth
hritis of herr hands. Heer has hand
d Xrays
donne.

Whiich of the following X ray cha anges suggests rheum matoid arth hritis instea
ad of a
seroonegativeaarthropathy?
A.Osteo
osclerosis
B.Osteo
ophytes
C.Osteo
oporoticchan
nges
D.Periarticularerosions
E.Losso
ofjointspacee

Answer:d)perriarticularerrosions.

Osteeophytesanndlossofjo
ointspaceaarecommon nlyfoundinnosteoarthritis,althou
ughthey
can alsobefou
undinrheu umatoidartthritis.Periaarticularero
osionsarem
mostsuggeestiveof
rheu
umatoidartthritis.


Periiarticularerrosions

Dr.K
KhalidYusuffElzohrySo
ohagTeachin
ngHospital2012
48
www.MRCPass.com

Rheumatology

RheumatologyQ045

A 76 year old woman presents with weakness of his hand. On examination there was
tenderness, crepitus and bony swellings over the base of the first metacarpal and
wastingoftherightthenareminence.
InvestigationsrevealanESRof25mm/1sthr,aCRPof10mg/L,aUrateconcentrationof
0.42 (0.190.36). Her Rheumatoid factor was 60 IU/L (<30). An xray of the right hand
showed a loss of joint space, periarticular sclerosis and osteophytes of the first
carpometacarpaljoint.

Whatisthelikelydiagnosis?
A.Osteoarthritis
B.DeQuervainstenosynovitis
C.Pseudogout
D.Gout
E.Rheumatoidarthritis

Answer:a)osteoarthritis.

Thepatienthasclinicalandradiologicalfeaturesconsistentwithosteoarthritis(OA)of
the1strightcarpometacarpal(CMC)joint.Theconditionischaracterisedbyjointpain,
crepitus and stiffness after movement. Joint swellings are bony in nature (Bouchard's
and Heberden's nodes), unlike boggy swellings which occurs in inflammatory arthritis.
This patients inflammatory markers are mildly raised only, making an inflammatory
arthritisunlikely.

Dr.KhalidYusufElzohrySohagTeachingHospital2012
49
www

w.MRCPass.ccom Rheum
matology

RheumatollogyQ0466

A700yearoldm manpresen ntswithsevverebackpains.AnHisstotalserumproteiniis85g/l


withhanalbumiinof41g/l.
A chest X rayy shows sevveral lucen dies. A sterrnal bone marrow
ncies in verrtebral bod
aspiirateobtainnsadarkred djellylikem
materialinthesyringe.

Thesmearofa aspirateislikkelytoshowwwhichpro ominentfea atures?


A.Macrophages
B.Osteo
oblasts
C.Plasm
macells
D.Pneumocytes
E.Osteo
osarcoma

Answer:c)Plassmacells.

Thee diagnosis is multiple myeloma. The bone marrow neeedle is likeely to be in


n a lytic
lesio
on filled with
w plasmaa cells. Hiss serum glo
obulins aree high from
m the mon noclonal
gammmopathy.


Apllasmacell

Dr.K
KhalidYusuffElzohrySo
ohagTeachin
ngHospital2012
50
www.MRCPass.com

Rheumatology

RheumatologyQ047

A75yearoldmanpresentswithbilateralhippains.Investigationsreveal:
Correctedcalcium2.5(2.22.6)mmol/l
ESR22mm/1sthr
Alkalinephosphatase800iu/L(50100)
gammaGT22iu/L

Whatisthelikelydiagnosis?
A.Osteoporosis
B.Osteomalacia
C.Paget'sdiseaseofthepelvis
D.Polymyalgiarheumatica
E.Multiplemyeloma

Answer:c)Paget'sdiseaseofthepelvis.

This elderly patient presenting with bone pains has significantly elevated alkaline
phosphatasebutnormalcalciumconcentrationssuggestingadiagnosisofPaget's.

Dr.KhalidYusufElzohrySohagTeachingHospital2012
51
www

w.MRCPass.ccom Rheum
matology

RheumatollogyQ0488

A 25 year old Turkish maan was noteed by ophtthalmologistts to have a posteriorr uveitis
requuiringhighdoseimmunosuppresssion.Hehassahistoryo ofrecurrenttmouthulccersand
painnfululcersoonthescrottum.

Wha atisthelikeelydiagnosiis?
A.Behceet'ssyndrom
me
B.Reiterssyndromee
C.Ankylosingspondylitis
D.Takayyasu'sarterittis
E.Giantcellarteritiss

Answer:a)Beh
hcet'ssyndrrome.

Behhcet's syndrome is claassically chharacterized d as a triad of symptoms that include


recuurringcropssofmouth ulcers(calledapthousulcers),genitalulcerrs,anduveiitis.The
ulceersareusuaallypainful.Thediseasseismoreffrequentan ndseverein npatientsfrromthe
EastternMediteerraneanan ndAsiathan nthoseofEEuropeandeescent.
Eye manifestattions may result
r in bliindness. In addition, iritis, retinaal vessel occclusions
andopticneurritiscan be found. Hyp popyonuve eitis(pus in
ntheanteriorchamberofthe
eye), which is
i considerred the haallmark off Behets disease, is i in fact a rare
man nifestation. ThearthrittisofBeheetsdiseaseeisusually intermitten nt,selflimitted,not
defoormingandlocalizedto othekneessandankless.


Oralulceration
ninBehcet'ssdisease

Dr.K
KhalidYusuffElzohrySo
ohagTeachin
ngHospital2012
52
www.MRCPass.com

Rheumatology

RheumatologyQ049

A70yearoldwomanhaspolydipsiaandpolyuriafor3months.Shealsocomplainsof
loinpains.Shehasnotbeenonanymedication.Investigationsreveal:
serumurea6(2.57.5)mol/l
serumcreatinine80(60110)mol/l
serumalbumin38g/L(3749)
serumtotalcalcium3.1(2.22.6)

Whatisthelikelycauseofthehypercalcaemia?
A.Multiplemyeloma
B.Sarcoidosis
C.Paget'sdisease
D.Primaryhyperparathyroidism
E.VitaminDtoxicity

Answer:d)Primaryhyperparathyroidism.

PrimaryhyperparathyroidismiscausedbyanoverproductionofPTH.
Excess PTH results in an increase in bone breakdown by means of osteoclastic
resorption with subsequent fibrous replacement and reactive osteoblastic activity.
Historically,inclassicprimaryhyperparathyroidism,nephrolithiasiswasnotedin50%of
patients, and it was the most common clinical presentation of the disease. Additional
manifestations of primary hyperparathyroidism include pancreatitis, peptic ulcer
disease,andcardiovascularabnormalities.

Dr.KhalidYusufElzohrySohagTeachingHospital2012
53
www.MRCPass.com

Rheumatology

RheumatologyQ050

A42yearoldladypresentswithbackache.Herbloodresultsareasfollows:
Hb11g/dL
Ca1.9mmol/L
Phosphate0.8mmol/L
Alkalinephosphatase220U/L
Albumin38g/L
Urea7mmol/L
Sodium142mmol/L
Potassium3.9mmol/L

Whatisthediagnosis?
A.Osteoporosis
B.Paget'sdisease
C.Osteomalacia
D.Xlinkedhypophosphataemicrickets
E.Hypoparathyoroidism

Answer:c)osteomalacia.

The patient has osteomalacia with secondary hyperparathyroidism causing low


phosphate levels. Osteomalacia is due to vitamin D deficiency which could be due to
malabsorptionordietarydefiency,orrenal/liverdisease.

Dr.KhalidYusufElzohrySohagTeachingHospital2012
54
www.MRCPass.com

Rheumatology

RheumatologyQ051

A 32 year old woman has a deep vein thrombosis. Her previous history included
investigationsformiscarriages.
Investigations revealed: Haemoglobin 11.9 g/dl, White cell count 4 x 10^9/L, Platelet
count30x10^9/L.

Whichofthesearelikelytobeabnormal?
A.Homocysteinelevel
B.ANCA
C.ProteinC
D.Antiphospholipidantibody
E.Coomb'stest

Answer:d)Antiphospholipidantibody.

Antiphospholipid syndrome leads to venous and arterial thrombosis, livedo reticularis,


splinter hemorrhages, leg ulcer, multiinfarct dementia, chorea, Thrmobocytopenia
(40%ofpatients),hemolyticanemiaandlatetermmiscarriages.

Dr.KhalidYusufElzohrySohagTeachingHospital2012
55
www.MRCPass.com

Rheumatology

RheumatologyQ052

A 35 year old man presents with acute stiffness swelling of his knees and ankles, a
painful rash on his legs. The ESR was 100 mm/hour. Chest Xray showed hilar
lymphadenopathybilaterally.

Whatisthelikelyprogressionofthearthralgia?
A.Chronicarthritis
B.Septicarthritis
C.Improvementonlywithsteroids
D.Spontaneousimprovement
E.Permanentjointdestruction

Answer:d)spontaneousimprovement.

Thedescriptiontypicalofacutesarcoidosiserythemanodosum,oligoarthropathyand
hilar lymphadenopathy. This usually has a good prognosis, with resolution over 68
weeks.

Dr.KhalidYusufElzohrySohagTeachingHospital2012
56
www.MRCPass.com

Rheumatology

RheumatologyQ053

A75yearoldladypresentswithbackpains.LateralspineXraysandpelvicxraysshow
osteopenia. A serum corrected calcium is 1.8 mmol/l and phosphate is 0.6 mmol/l.
Alkalinephosphataseis360U/l.

Whichdiagnosisismostlikely?
A.Myeloma
B.Osteoporosis
C.Osteomalacia
D.Paget'sdisease
E.Ankylosingspondylitis

Answer:c)osteomalacia.

Osteomalacia is more likely than osteoporosisdue to the low calcium, low phosphate
andraisedalkalinephosphatse.TheconditioniscausedbylowvitaminDlevels.
Alkalinephosphataseisraisedwhenthereisincreasedosteoblasticactivity,conditions
itmayberaisedin:
Paget'sdisease
osteomalaciaandrickets
renalosteodystrophy
bonemetastases

Dr.KhalidYusufElzohrySohagTeachingHospital2012
57
www.MRCPass.com

Rheumatology

RheumatologyQ054

A 22 year old lady has a sw inging fever, half a stone weight loss over 2 months,
generalised myalgia, polyarthralgia affecting wrists, knees, ankles, elbows
metacarpophalangealjointsandasorethroat.
Investigationsshow:Hb9.5g/l,MCV85fl,ESR92mminfirsthour,CRP45g/l,serum
ferritin1600mg/dl,RFnegative,ANAnegative,ENAnegative,ASOtitre<200iu.

Whatisthelikelydiagnosis?
A.Rheumatoidarthritis
B.Ankylosingspondylitis
C.AdultonsetStill'sdisease
D.Inclusionbodymyositis
E.Polymyositis

Answer:c)AdultonsetStill'sdisease.

AdultOnsetStillsDisease(AOSD)isan acutefebrileillness inyoungadults.Itusually


affectsmultipleorgans,butisadiagnosisofexclusion.
Clinical features include a high fever, arthralgia and arthritis, phayngitis, typical rash
(evanescent salmoncolored, macular or maculopapular eruption), lymphadenopathy,
andserositis.Chronicarthritisandconstitutionalsymptomsarecommon.
The triad of fever, rash, and arthralgia are often absent during the first month of the
illness.Theusualjointsaffectedarewrists,knees,andanklesindescendingorder.
Twothirdsofcasesexperiencepolyarticulararthritisandonethirdhavemonoarticular
symptoms.
Approximately 1/3 of patients have chronic persistent disease with progressive joint
damage.

Dr.KhalidYusufElzohrySohagTeachingHospital2012
58
www.MRCPass.com

Rheumatology

RheumatologyQ055

A65yearoldwomanhasaswollen,erythematouskneejoint.Aspirationofthejointwas
performed.Microscopyshowedpositivelybirefringentcrystals.

Whatarethecrystalscomposedof?
A.Calciumcarbonate
B.Magnesiumsulphate
C.Urate
D.Calciumpyrophosphate
E.Aminolaevulinicacid

Answer:D)calciumpyrophosphate.

Pseudogout is caused by the deposition of calcium pyrophosphate deposition rather


than the deposition of uric acid derivatives that cause gout. Also, in pseudogout,
synovialfluidsamplesobtainedwithaspirationhavepositivebirefringence.Thisfinding
isindirectcontrasttothenegativebirefringenceingout.

Dr.KhalidYusufElzohrySohagTeachingHospital2012
59
www.MRCPass.com

Rheumatology

RheumatologyQ056

A45yearoldwomanpresentswithclaudicationinherlowerlimbs.Sheisnotedtohave
absentleftarmpulses.
Thepreviousyearshehadasmallhemisphericcerebrovascularinfarct.

Whatisthelikelydiagnosis?
A.Giantcellarteritis
B.Takayasu'sarteritis
C.Familialhypercholesterolaemia
D.Coarctationoftheaorta
E.Antiphospholipidsyndrome

Answer:b)Takayasu'sarteritis.

Takayasu'sarteritisisalargevesselvasculitisofunknownorigin.Thevasculiticprocess
involves structures such as the aorta, great vessels, the sclera and the cardiac
conductiontissues.
Women are affected more than men, usually in the second and third decades of life.
Presentation is often with symptoms such as fever, weight loss, night sweats and
arthralgias. Symptoms related to ischaemia may include ischaemic stroke, visual
disturbancesandclaudication.

Dr.KhalidYusufElzohrySohagTeachingHospital2012
60
www.MRCPass.com

Rheumatology

RheumatologyQ057

A 70 year old man develops weakness of the shoulders and around the hips over a 6
month period. He hasalso noticed weakfinger flexors. He complained of difficulty sw
allow ing liquids. There is no other significant past medical history. He smokes 15
cigarettesadayanddrinksabottleofwineattheweekend.Acreatininekinaselevel
comesbackat120U/l.
A muscle biopsy sample shows myopathic changes. There are also inflammatory
infiltratesandintracytoplasmicvacuolespresent.

Whatisthelikelydiagnosis?
A.Polymyositis
B.Fibromyalgia
C.Polymyalgiarheumatica
D.Dermatomyositis
E.Inclusionbodymyositis

Answer:e)inclusionbodymyositis.

The diagnosis is inclusion body myositis (IBM). This is an inflammatory condition


affecting patients over the age of 50. Proximal muscles in the upper and lower limbs,
andfingerflexorsarepredominantlyinvolved.TheonsetofmuscleweaknessinIBMis
generallygradual(overmonthsoryears).Dysphagiaiscommon,occurringin4066%of
patients.
IBMoccursmorefrequentlyinmenthanwomen.CKmaybenormal.Amusclebiopsy
usuallyshowsintracytoplasmicinclusionsandalsoinflammatoryinfiltrates.

Dr.KhalidYusufElzohrySohagTeachingHospital2012
61
www

w.MRCPass.ccom Rheum
matology


Thigghatrophysseenininclusionbodymyositis


Inclusionbodie
esandinflam
mmatoryin
nfiltrates(arrrows)

Dr.K
KhalidYusuffElzohrySo
ohagTeachin
ngHospital2012
62
www

w.MRCPass.ccom Rheum
matology

RheumatollogyQ0588

A 53
5 year old d woman with
w rheummatoid arthhritis was rreferred with iron deeficiency
anaemia. She had exten nsive investtigations. Endoscopy
E showed gaastritis andd antral
erossions, small bow el biopsy shoowing mild villous bluunting, andd apopotic bodies,
Colo
onoscopyw wasnormal.

Wha atisthecau
useoftheseefindings?
A.Meth
hotrexate
B.Azath
hioprinetherrapy
C.Crohn
nsdisease
D.NSAID
Dtherapy
E.Felty'ssyndrome

Answer:d)NSA
AIDtherapyy.

TheeendoscopyyfindingsarresuggestivveofNSAID
Drelatedgasstritis,whicchisthemo
ostlikely
causeofirondeficiencyan
naemia.


NSA
AIDrelatedgastritis

Dr.K
KhalidYusuffElzohrySo
ohagTeachin
ngHospital2012
63
www.MRCPass.com

Rheumatology

RheumatologyQ059

A 45 year old woman complains of 12 months of mid and lower back pain associated
withstiffnessinhershoulders,wrists,smalljointsofthehands,hipskneesandfeet.The
jointstiffnessismorepronouncedinthemorningandlasts23hoursandisassociated
withseverefatigue.
She also experiences occasional pins and needles affecting all of the right hand and
frequentfrontalandtemporalheadache.
Clinicalexaminationdemonstratesnosynovitisbuttendernessaroundthebaseofthe
cervical spine, across the shoulders, over the costochondral cartilages, greater
trochanterandtheknee.
Investigationsareasfollows:CRP<5g/l,ESR20mminthefirsthour,CK120U/l,serum
immunoglobulinsandproteinelectrophoresisnormal.

Whatisthediagnosis?
A.SLE
B.Polymyalgiarheumatica
C.Fibromyalgia
D.Dermatomyositis
E.Polymyositis

Answer:c)Fibromyalgia.

Theabovesymptomsareallcharacteristicforfibromyalgia.Theinflammatorymarkers
andCKarenotraised,makingtheotherdiagnosesunlikely

Dr.KhalidYusufElzohrySohagTeachingHospital2012
64
www.MRCPass.com

Rheumatology

RheumatologyQ060

A35yearoldmanpresentswithhotswollenelbowjoint.Hehasatemperatureof37C
andCRPof350mg/l.Adiagnosisofsepticarthritisissuspected.

Whichofthefollowingorganismsismostcommonlyisolatedfromjoints?
A.Staphyloccocusaureus
B.Neisseriagonorrhoeae
C.haemolyticstreptococcus
D.Pseudomonasaeruginosa
E.Moraxellacatarrhalis

Answer:a)staphyloccocusaureus.

Staphaureusisthecommonestorganism(over50%).Neisseriagonorrheacanoccurin
patients with sexually transmitted disease. haemolytic streptococci can cause
impetigo,sorethroatandrheumatic fever.Moraxella isagram negativecoccuswhich
cancausepneumoniainCOPDpatients.

Dr.KhalidYusufElzohrySohagTeachingHospital2012
65
www.MRCPass.com

Rheumatology

RheumatologyQ061

A 66 year old lady had a fall and fractured femur. A DEXA scan shows reduced bone
mineraldensityconsistentwithosteoporosis.

Whattreatmentshouldbegiventoreducethelikelihoodoffuturefractures?
A.Alendronate
B.Alendronateandcalcium
C.Alendronate,calciumandvitaminD
D.Calcium
E.VitaminD

Answer:C)alendronate,calciumandvitaminD

Any patient above the age of 65 with osteoporosis is recommended to be on


bisphosphonates.Inaddition,thispatienthassustainedafracture,andshouldalsobe
oncalciumandvitaminD(calcichewD3)aswell.

Dr.KhalidYusufElzohrySohagTeachingHospital2012
66
www.MRCPass.com

Rheumatology

RheumatologyQ062

A82yearoldwomanpresentswithconfusion.OfnoteinherpasthistorywasRaynauds
phenomenon.
Herinvestigationsshow:
haemoglobin9.7g/dl
whitecellcount3.6x10^9/l
plateletcount100x10^9/l
serumtotalprotein120g/l
serumimmunoglobulins:IgA0.75g/l(0.83),IgG16g/l(613),IgM35g/l(0.42.5)

Whichofthefollowingcomplicationsisshelikelydevelop?
A.Urinarytractinfection
B.Hyperviscositysyndrome
C.Pathologicalbonefracture
D.Acuterenalfailure
E.Erythemanodosum

Answer:b)Hyperviscositysyndrome.

ThelikelydiagnosisisWaldenstromsMacroglobulinaemia(WM).
Waldenstroms macroglobulinaemia is a type of nonHodgkins lymphoma. It is a
condition which typically presents in the seventh and eighth decade of life. It is
characterized by the presence of a high level of a macroglobulin immunoglobulin M
[IgM]andelevatedserumviscosityinthepresenceofalymphoplasmacyticinfiltratein
thebonemarrow.Thetreatmentischemotherapy(ChlorambucilorFludarabine).

Dr.KhalidYusufElzohrySohagTeachingHospital2012
67
www.MRCPass.com

Rheumatology

RheumatologyQ063

A 72 year old woman complained of pain at the base of her right thumb. There was
tenderness and swelling of the right first carpometacarpal joint. Finkelsteins test is
negative.

Whatisthemostlikelydiagnosis?
A.Avascularnecrosisofthescaphoid
B.DeQuervain'stenosynovitis
C.Osteoarthritis
D.Psoriaticarthritis
E.Rheumatoidarthritis

Answer:C)osteoarthritis.

The tenderness at the carpometacarpal joint is most likely due to osteoarthritis in a


patientofthisage.

Dr.KhalidYusufElzohrySohagTeachingHospital2012
68
www

w.MRCPass.ccom Rheum
matology

RheumatollogyQ0644

A 45
4 year old man presents to thee opthalmo
ologist and was found
d to have anterior
a
uveitis.Uponeenquiry,hehasmouthulcersandarashonhisleg.

Wha atisthelikeelydiagnosiis?
A.ALam
myloidosis
B.Familialmediterraaneanfever
C.Behceet'sdisease
myalgiarheumatica
D.Polym
E.SLE

Answer:c)Beh
hcet'sdiseasse.

Beh
hets disease may cause antterior or posterior uveitis, co orneal ulceration,
conjjunctivitis, papillitis, SVC and IVC
I thromb
bosis, pulm
monary em mbolism, errythema
nod
dosum,pusttularlesionss,acneiform mnodules,p
pathergyanndoralulceration.


AnteriorUveitisinBehcet'sdisease

Dr.K
KhalidYusuffElzohrySo
ohagTeachin
ngHospital2012
69
www.MRCPass.com

Rheumatology

RheumatologyQ065

InapatientwithSLE,theriskofcardiovascularabnormalityisincreasedwithpresence
ofwhichoneofthefollowing?
A.Anticardiolipinantibody
B.AntiRoantibody
C.AntiLaantibody
D.ANAantibody
E.AntiJoantibody

Answer:B)antiRoantibody

AntiRoisimportantinpregnancysinceitisassociatedwithbabiesbornwithcongenital
heartblock.

Dr.KhalidYusufElzohrySohagTeachingHospital2012
70
www.MRCPass.com

Rheumatology

RheumatologyQ066

A50yearoldsecretarynoticedtinglingandnumbnessover thepalmarsurface ofher


thumb,indexandmiddlefingersafterseveralhoursathercomputer.Paininthesame
areasoftenoccuratnight.

Whatisthediagnosis?
A.Rheumatoidarthritis
B.Gout
C.Dermatomyositis
D.Carpaltunnelsyndrome
E.Psoriaticarthritis

Answer:d)Carpaltunnelsyndrome.

The diagnosis is carpal tunnel syndrome, and the symptoms are due to tenosynovitis
whichisworsenedbyrepetitivestrainimposedbytyping.

Dr.KhalidYusufElzohrySohagTeachingHospital2012
71
www.MRCPass.com

Rheumatology

RheumatologyQ067

A75yearoldmancomplainsofpainandstiffnessinbothhisshoulders.Hehaslost1/2
a stone in last 6 weeks due to loss of appetite. Investigations show : ESR 95 mm/hr,
normochromicnormocyticanaemiaandapositiverheumatoidfactor.

Thelikelydiagnosisis:
A.Fibromyalgia
B.Polymyositis
C.Dermatomyositis
D.PolymyalgiaRheumatica
E.Chronicmyeloidleukaemia

Answer:d)PolymyalgiaRheumatica.

PolymyalgiaRheumaticaisassociatedwithweightloss,anemia&malaise.
It can also be associated with a false positive rheumatoid factor, especially in the
elderly.

Dr.KhalidYusufElzohrySohagTeachingHospital2012
72
www.MRCPass.com

Rheumatology

RheumatologyQ068

A12yearoldboyhasfevers.Hehashadelbow,shoulderandhipandkneepainsover
the last few months. On examination, he has a temperature of 37.5 C and palpable
hepatosplenomegaly.
InvestigationsshowaCRPof30andnegativeANA.

Whatisthelikelydiagnosis?
A.Marfan'ssyndrome
B.EhlerDanlossyndrome
C.Perthe'sdisease
D.Juvenilechronicarthritis
E.Osteoarthritis

Answer:d)juvenilechronicarthritis.

Juvenile chronic arthritis (JCA) is a form of seronegative arthritis in the young (adult
formknownasStillsdisease)whichisrheumatoidfactorandANAnegative.Diagnostic
criteria include high fever, hepatomegaly, splenomegaly, lymphadenopathy, serositis
(pleuritis,pericarditis),leucocytosis.Bonedestructionandmicrognathiaoccurs.

Dr.KhalidYusufElzohrySohagTeachingHospital2012
73
www.MRCPass.com

Rheumatology

RheumatologyQ069

A25yearoldladypresentstoclinicwithapainfullyswollenhotrightknee.Shegivesa2
week history of migratory polyarthritis and urethritis. Clinical examination reveals a
pustular skin rash, right Achilles tendinitis and left plantar fasciitis. She is HLAB27
negative.

Whatisthemostlikelydiagnosis?
A.Relapsingpolychondritis
B.Reiterssyndrome
C.Gonoccocalarthritis
D.Nonspecificurethritis
E.Behcet'ssyndrome

Answer:c)Gonoccocalarthritis.

Gonoccocalarthritisclassicallypresentswithahotjointonabackgroundofamigrating
polyarthropathy. It affects w omen more frequently than men (4:1) and its highest
incidence is among sexually active adolescent girls. There is also increased risk during
menstruationandpregnancy.
Twoformsofarthritisexistonewithskinrashesandmultiplejointinvolvement,anda
second,lesscommon,forminwhichdisseminatedgonococcemialeadstoinfectionofa
singlejoint(monoarticular)andjointfluidculturesarepositive.

Dr.KhalidYusufElzohrySohagTeachingHospital2012
74
www.MRCPass.com

Rheumatology

RheumatologyQ070

A50yearoldwomanwithlongstandingrheumatoidarthritis,andasiccasyndrome.She
presentswithincreasingmalaise.Investigationsshowed:
Hb7.5g/dl
Neutrophilcount1.2x10^9/l
plateletcount90x10^9/l
HerESRwas120mm/hrandCRP145mg/dl
Onexamination,shewaspaleandhadpalpablesplenomegalyof6cmedge.

Themostlikelycauseofherhaematologicabnormalitiesis:
A.Anaemiachronicdisease
B.UpperGIbleed
C.Felty'ssyndrome
D.Folatedeficiency
E.Irondeficiency

Answer:c)Felty'ssyndrome.

Felty'ssyndromeisthetriadofseropositivearthritis,splenomegalyandneutropenia.
The cause of Felty's syndrome is not known, but is most commonly associated with
rheumatoidarthritis.

Dr.KhalidYusufElzohrySohagTeachingHospital2012
75
www.MRCPass.com

Rheumatology

RheumatologyQ071

32 year old lady upper middle class white lady who used to be an athlete has been
referred for investigation due to tiredness. She mentions fatigue which is chronic and
occurs even after minor physical w ork. This has been going on for 3 years.
Investigations including CK, ESR, EMGs and muscle biopsy have revealed no obvious
medicalcauseforthis.

Whichofthefollowingisthebesttreatment?
A.Erythropoietininjections
B.Cognitivebehaviouraltherapy
C.Gradedexerciseprogramme
D.Codeine
E.Fluoxetine

Answer:C)Gradedexerciseprogramme.

ChronicFatiguesyndromeisdefinedbysymptomsandnotsigns.Theclinicalprofileof
an individual with CFS is of a highachieving student or athlete who usually is female
(80%), white, and middleclass to upper middleclass. Treatment is largely supportive
andresponsivetosymptomatology.Thisincludesphysicaltherapyandmodestaerobic
oranaerobicexercise(ifpossible)toavoidcardiovasculardeconditioning.Sleepmaybe
addressed with medication; often, melatonin or nighttime amitriptyline is helpful. If
presentandsevere,painoftenisaddressedinapainclinic.

Dr.KhalidYusufElzohrySohagTeachingHospital2012
76
www

w.MRCPass.ccom Rheum
matology

RheumatollogyQ0722

A45
5yearoldp
patientpressentswithpproximalmuscleweakness,particcularlyinth
helower
limb
bs.Shehasaheliotropicrasharou
undtheeyeesandalsoGGottronspapules.

Whiichoneofth
hefollowing
gantibodieesismoststtronglyasso
ociated?
A.La
B.Ro
C.Jo1
D.SCL7
70
E.AntiD
DsDNA

Answer:C)Jo1
1.

Thee diagnosis is dermato omyositis. Anti Jo1 antibody


a is associated with acute onset
myoositis, particularly dermatomyositis. The limmb girdle or proximal muscles arre most
seveerelyaffectedinbothp polymyositiisanddermmatomyositis.


Gotttron'spapu
ules

Dr.K
KhalidYusuffElzohrySo
ohagTeachin
ngHospital2012
77
www.MRCPass.com

Rheumatology

RheumatologyQ073

A9yearoldboyisbroughttothepaediatricianwithcomplaintsoffeverfor1week.He
alsohascrackedlips,whicharepainful.
Onexamination,hiseyesareredandhehasalymphnodeinthecervicalregion.Kaw
asakisyndromeissuspected.

Whichofthefollowingisthebesttreatmentchoice?
A.Intravenousimmunoglobulim
B.Hydrocortisone
C.Abxicimab
D.Amoxicillin
E.Lowmolecularweightheparin

Answer:a)Intravenousimmunoglobulin.

ThisisacaseofKawasakissyndrome,whichisalsoknownasmucocutaneouslymph
node syndrome and occurs mainly in children under 10 years of age. It is a form of
vasculitis, which affects coronary arteries and is associated with the development of
coronaryaneurysms.
AspirinandIVimmunoglobulinathighdosesisthetreatmentofchoice.Corticosteroids
used to be contraindicated, but recently trials have shown that it reduced the risk of
heartdamagecausedbycoronaryvesselvasculitis.
Infliximabisalsobeingusedintrials.Warfarinisusedifcoronaryaneurysmsdevelopas
acomplication.

Dr.KhalidYusufElzohrySohagTeachingHospital2012
78
www.MRCPass.com

Rheumatology

RheumatologyQ074

A55yearoldwomanpresentswitha2monthhistoryofpainaffectingthecervicalspine
and both shoulders, this was preceded by pain in the lower back and both hips. Early
morning stiffness lasts until lunchtime and she feels markedly tired and weak.
Examinationrevealsbilateralkneeeffusionsandarightcarpaltunnelsyndrome.
InvestigationsdemonstrateanormochromicnomocyticanaemiaofHb9.8g/dl,ESR72
mm in the first hour, CRP 12 g/l, serum immunoglobulins and protein electrophoresis
show a polyclonal increase in gamma globulins and elevated alpha 1 and alpha 2
globulinsbutnoparaproteinband.

Whatisthemostlikelydiagnosis?
A.Rheumatoidarthritis
B.Paraneoplasticsyndrome
C.Mixedconnectivetissuedisease
D.Scleroderma
E.Polymyalgiarheumatica

Answer:e)Polymyalgiarheumatica.

Polymyalgia rheumatica (PMR) is a clinical diagnosis based on pain and stiffness of


pelvicgirdleandshoulder.Itismorecommonaftertheageof55.
Incontrasttopolymyositisthereisnomuscularweakness.Earlymorningstiffnessofthe
hipandshouldergirdlesiscommon.Anormochromicnormocyticanaemiaisassociated.

Dr.KhalidYusufElzohrySohagTeachingHospital2012
79
www.MRCPass.com

Rheumatology

RheumatologyQ075

A55yearoldmanpresentswithsuddenonsetofarightsidedfootdropwithnumbness
overthedorsumofhisrightfoot.Hehasweaknessofdorsiflexionandeversionofhis
rightfoot.Thereisanareaofdiminishedsensationoverthedorsumoftherightfoot.He
also has a left wrist drop with loss of sensation over the dorsal aspect of the first
interosseusspace.Forthelasttwomonths,hehaslostweightandhehadgeneralized
myalgia.
Investigationsreveal:
Hb11.5g/dL
WBC13x10^9/l
Neutrophils9.2x10^9/l
Lymphocytes2.2x10^9/l
ESR60mm/1sthour
Urinalysis:Protein++,Blood++

Themostlikelydiagnosisis:
A.Tuberculosis
B.Polyarteritisnodosa
C.Sarcoidosis
D.Rheumatoidarthritis
E.Systemiclupuserythematosus

Answer:b)Polyarteritisnodosa.

The patient has a systemic illness with mononeuritis multiplex and renal involvement.
Although all the listed conditions can cause mononeuritis, PAN is one of the few
connective tissue disorders that usually occur in middleaged men unlike RA and SLE,
whicharecommonerinfemales.

Dr.KhalidYusufElzohrySohagTeachingHospital2012
80
www

w.MRCPass.ccom Rheum
matology

RheumatollogyQ0766

A600yearoldmmanpresen ntswithpaininhislefttfoot.Hem
mentionsthaathehashadpoor
mob bility.Generalexaminaationrevealsaswollen nankle.Neu urologicalexxaminationreveals
abseentanklejeerkandweaakfootflexion/extensiiononthelleft.HisESR Ris30mm//hr,CRP
is<5
5mg/l.Jointtxrayreveealssubchonndralfractu ngandanarrowed
ures,softtisssueswellin
jointspace.

Thelikelydiagnnosisis:
A.Charccot'sjoint
B.Osteo
oarthritis
C.Gout
nilechronicaarthritis
D.Juven
E.Previo
ousroadtraffficaccident

Answer:a)Chaarcot'sjoint.

Chaarcot joint or neurop pathic joint disease caan be caussed by diaabetic neurropathy,
syphhillis, syringgomyelia or
o leprosy. The X ray changes are
a reduced d joint space with
periiarticular destruction,
d , subchonddral bodiess, loose boodies and occasionally joint
dislo
ocation.


Chaarcot'sjoint

Dr.K
KhalidYusuffElzohrySo
ohagTeachin
ngHospital2012
81
www.MRCPass.com

Rheumatology

RheumatologyQ077

A55yearoldmanpresentswithlethargy,polyuria,polydipsiaandstiffnessofthehands.
Hehasevidenceofanarthopathyaffectingthe2nd&3rdmetacarpophalangealjointsof
bothhands.Xrayconfirmsevidenceofdegenerativediseaseatthesesites.

Whichofthefollowingthelikelydiagnosis?
A.Behcet'ssyndrome
B.Haemochromatosis
C.Amyloidosis
D.Reiter'ssyndrome
E.Osteoarthritis

Answer:b)haemochromatosis.

The characteristic feature of haemochromatotic arthropathy is involvement of the


secondandthirdmetacarpals.Thearthropathycanleadtoextensivejointdestruction.
These patients may have episodes of acute, inflammatory pseudogout from such
deposition.

Dr.KhalidYusufElzohrySohagTeachingHospital2012
82
www.MRCPass.com

Rheumatology

RheumatologyQ078

An22yearoldgirlpresentswitha6weekhistoryofpolyarthralgiawithearlymorning
stiffness.HersymptomsrespondedwellinitiallytoIbuprofenbutshethentheyworsen
again. She is otherw ise well apart from a history of acne which well controlled on
Minocycline.Hermothersevererheumatoidarthritis.
Investigations show : ESR 40 mm/hr, CRP 110 mg/l, rheumatoid factor negative, ANA
stronglypositive(1:1600),antidsDNAantibodiesnegative.

Whatisthelikelycauseofhersymptoms?
A.Druginducedlupus
B.SystemicLupusErythematosus
C.Psoriaticarthropathy
D.Juvenilechronicarthritis
E.Fibromyalgia

Answer:a)Druginducedlupus.

The likely diagnosis is drug induced SLE. Minocycline is one of the causes well
documentedasacauseofdruginducedSLE.
Characteristically,inflammatorymarkerssuchasESR&CRParebothmarkedlyelevated.
ANAcanbestronglypositivebutAntidsDNAantibodiesareusuallynegative.Symptoms
usuallyresolvefollowingwithdrawalofthedrugafterseveralmonths.

Dr.KhalidYusufElzohrySohagTeachingHospital2012
83
www.MRCPass.com

Rheumatology

RheumatologyQ079

A50yearoldwomanpresentstocasualtywitha2dayhistoryofpainandswellingof
theleftankle.Shedeniesanyhistoryofrecenttrauma.Onexamination,shewasfebrile,
temperature38.3C.Theleftanklewasswollenandverytenderwithareducedrange
ofmovement.

Whichofthefollowinginvestigationswouldbemosthelpful?
A.Uratelevel
B.ESR
C.Rheumatoidfactor
D.Aspirationofsynovialfluid
E.KneeXray

Answer:d)aspirationofsynovialfluid.

Joint fluid examination is important in excluding septic arthritis, and can also help to
confimgoutorpseudogout.

Dr.KhalidYusufElzohrySohagTeachingHospital2012
84
www.MRCPass.com

Rheumatology

RheumatologyQ080

A 35 year old woman with newly diagnosed rheumatoid arthritis enquires about risk
factors.

Whichoneofthefollowingisariskfactorforrheumatoidarthritis?
A.Pneumococcalinfection
B.HLADR4
C.SLE
D.HIV
E.Renalfailure

Answer:b)HLADR4.

Riskfactorsforrheumatoidarthritisare:
HLADR4
EBV
parvovirusB19andrubellainfections
bloodtransfusion
smoking(inducesRFproduction)
stress
obesity
PregnancyandOCPareprotective.

Dr.KhalidYusufElzohrySohagTeachingHospital2012
85
www.MRCPass.com

Rheumatology

RheumatologyQ081

A26yearoldmanpresentswitha6monthhistoryoflowbackpain.Thepainradiatesto
his buttocks. There is associated stiffness which is w orse in the morning and after
periodsofinactivity.

Whichofthefollowingsignsismostlikelytobepresent?
A.Footdrop
B.Positivefemoralstretchtest
C.PositiveTrendelenburgtest
D.Sacroiliacjointtenderness
E.Kernigssign

Answer:D)sacroiliacjointtenderness.

Thediagnosisisankylosingspondylitisasthisisayoungpatientwithlowerbackpains
andmorningstiffness.Sacroilitisisacommonmanifestation.

Dr.KhalidYusufElzohrySohagTeachingHospital2012
86
www

w.MRCPass.ccom Rheum
matology

RheumatollogyQ0822

A62
2yearoldm manhascom mplainsofaapainintheetemples,ffevers,sweaatsandmalaisefor
4w
weeks.Therrightsideoffthefacew
wasmildlysswollenand dheexperieencedvisuaallossin
therighteye.
Oneexamination,hehadp prominentaandtenderttemporalarrteriesonth heright.
Bloo
odtestsrevvealed:
Hb1
12.5g/dl MCV86fl
WCC12x10^9 9/l plattelets212xx10^9/l
ESRof90mm//hr
Whaatisthelikeelydiagnosiis?
A.Pituittarytumour
B.Gravee'sdisease
C.Pagett'sdisease
D.Catarract
E.Temp
poralarteritiss

Answer:e)tem
mporalarterritis.

Theediagnosisistemporalarteritis(giiantcellarte eritis).
Thee complications of Giaant Cell Artthritis are related to arterial infflammation
n. These
include:
blindness
absentpulsses
hypertensioon
angina
stroke
claudication


Prominenttem
mporalarterries

Dr.K
KhalidYusuffElzohrySo
ohagTeachin
ngHospital2012
87
www

w.MRCPass.ccom Rheum
matology

RheumatollogyQ0833

A600yearoldw womanpreesentswith ahistoryo ofprogressivedifficultyyinclimbin ngstairs


andrisingfrom mchairs.
Shealsocomplainsofdiffficultyinsw wallowing.Shecompllainsthatherhandsdiscolour
andbecomeco oldeasily.
On examinatio on she has purple disccolouration of her eyeelids and periorbital oedema.
o
Theereisweakn nessofthemmusclesofh herlimbgirrdles.

Wha atisthelikeelycauseoffthedysphaagia?
A.Oesophagealcanccer
B.Derm
matomyositis
C.SystemicLupusErrythematosu
us
D.Retro
osternalgoitrre
E.Sclero
oderma

Answer:b)Derrmatomyositis.

Thee diagnosis is dermato


omyositis. Dysphagia in dermato omyositis iss primarily due to
weaakness of the striated
d musculatuure of the posterior pharynx.
p Dyysphagia may
m also
resuult from cricopharyngeal obstrucction secondary to infflammation or fibrosiss of the
cricopharyngeu usmuscles.

Heliiotroperash
hindermattomyositis

Dr.K
KhalidYusuffElzohrySo
ohagTeachin
ngHospital2012
88
www.MRCPass.com

Rheumatology

RheumatologyQ084

A60yearoldmanisonfrusemideforlegoedema.Hepresentswithapainfulswollen
knee.Histemperatureis38Candhehasawhitecellcountof12x10^9/landCRPof
120mg/l.Uricacidis0.49mmol/l(<0.42).
Ajointaspiraterevealspuscellsandnegativelybirefringentcrystalswithpolarizedlight.
AftertreatmentwithNSAIDsfor48hourshehasnotimprovedandtheswellingpersists.
Therearenoorganismsculturedfromthejointaspirateorbloodcultures.

Whatisthebestcourseofaction?
A.Allopurinol
B.MRItolookforosteomyelitis
C.Intravenousflucloxacillin
D.Arthroscopyandwashout
E.Furtherjointaspirationwithdepomedroneinjection

Answer:e)furtherjointaspirationwithdepomedroneinjection.

This is a case of acute gout which has not settled on medical therapy (NSAIDs or
colchicine).Theraisedinflammatorymarkerscanbeduetoacutegout.Uricacidlevels
are notnecessarilyhigh in acutegout.Adepomedrone(steroid) injectionwithfurther
aspirationofjointfluidisthenextstep.

Dr.KhalidYusufElzohrySohagTeachingHospital2012
89
www.MRCPass.com

Rheumatology

RheumatologyQ085

A40yearoldladypresentswithpolyuriaandthirst.Herserumcalciumof2.85mmol/l
andaparathyroidhormoneof12(18)pmol/l.Shehasnormalrenalfunction.

Whichtestprovidesthebestassessment?
A.CTofherspine
B.DualenergyXrayabsorptiometry
C.VitaminDlevels
D.Isotopebonescan
E.Urinebencejonesprotein

Answer:b)DualenergyXrayabsorptiometry.

Thispatientislikely tohave primaryhyperparathyroidism(secondarytoaparathyroid


adenoma).Thebestassessmentistodeterminetheseverityoflossofbonedensitywith
aDEXAscaninordertoconsiderparathyroidsurgeryisnecessary.

Dr.KhalidYusufElzohrySohagTeachingHospital2012
90
www

w.MRCPass.ccom Rheum
matology

RheumatollogyQ0866

An445yearoldmanhashadworseniingbackpains,shouldeerpainsandrighthip painfor
8yeears.Thepaainistypicaallyworseaattheend ofday.He alsohasbo onyenlargem mentof
the distal inteerphalangeaal joints. An X ray off the shoullder reveals the preseence of
prominentosteeophytes.TThereissclerosisand narrowinggofthejoin ntspaceat thehip
jointsonthepe elvicXray.

Wha atisthelikeelydiagnosiis?
A.Osteo
omyelitis
B.Osteo
oarthritis
C.Osteo
omalacia
D.Rheumatoidarthritis
E.Pagett'sdisease

Answer:b)oste
eoarthritis.

Theeclinicalhisstorysuggesstsearlyon
nsetosteoarrthritis(whichcanbe idiopathico
oroccur
inaathletes).ThheXrayfeaaturesofosteophytes,sclerosisaandnarrow wingofjoinntspace
aresuggestiveofosteoartthritis.

Osteeophyteon
nashoulderrXray

Dr.K
KhalidYusuffElzohrySo
ohagTeachin
ngHospital2012
91
www.MRCPass.com

Rheumatology

RheumatologyQ087

A35yearoldwomanhasgeneralizedjointpainsandmuscleaches.Shealsocomplains
ofgrittysensationsintheeyeinthemornings.BloodtestsshowAntiNuclearAntibody
++,RheumatoidFactor++.

Whatisthediagnosis?
A.Polylmyositis
B.Reiterssyndrome
C.PrimarySjogrenssyndrome
D.Polyarteritisnodosa
E.Rheumatoidarthritis

Answer:C)PrimarySjogrenssyndrome.

Dry mouth, dry eyes, fatigue, muscle aches and joint pains are typical of Sjogrens
syndrome.80%ofpatientsareANApositiveand75%areRheumatoidfactorpositive.A
usefuldiagnostictestistheSchirmer'stest,whereapieceoffilterpaperisplacedinthe
corneroftheeyetomeasurethedegreeofwettingafterfiveminutes.

Dr.KhalidYusufElzohrySohagTeachingHospital2012
92
www.MRCPass.com

Rheumatology

RheumatologyQ088

A60yearoldwomanpresentswithatwoweekhistoryofmalaiseandlowerlimbjoint
pains. On examination she had a vasculitic rash over her shins, thighs and buttocks.
Investigationsrevealed:
Hb10.2g/dL
platelets265X10^9/L
creatinine380mol/L
antinuclearantibodiesNegative
antineutrophilcytoplasmicantibodiesNegative
antiglomerularbasementmembraneantibodiesNegative
dipstixurinalysisblood+++
protein+

Whatisthelikelydiagnosiscausingrenalimpairment?
A.Psoriaticarthritis
B.HenochSchonleinpurpura
C.Polymyositis
D.Membranousnephropathy
E.Myeloma

Answer:b)HenochSchonleinpurpura.

The distribution of the rash together with lower limb joint pains are suggestive of
HenochSchonleinpurpura.Thisusuallyoccursinchildrenaged210yearsbutcanoccur
in older age groups. The only w ay of differentiating this condition from other small
vesselvasculitidesisbybiopsy.ThiswouldshowIgAdepositioninvesselwallsondirect
immunofluorescence.

Dr.KhalidYusufElzohrySohagTeachingHospital2012
93
www.MRCPass.com

Rheumatology

RheumatologyQ089

A 28 year old woman presents with a right knee joint pain and a 4 month history of
weightloss.Shethinksshehaslostweightbecauseofdiarrhoea,whichoccursseveral
timesaday.Examinationrevealsaswollen,tenderrightkneejointwithasmalleffusion.

Thelikelydiagnosisis:
A.Reiter'ssyndrome
B.Inflammatoryboweldisease
C.Behcet'sdisease
D.Campylobacterinfection
E.Rheumatoidarthritis

Answer:b)Inflammatoryboweldisease.

The description of weight loss, diarrhoea and a mono/oligoarthropathy suggests a


diagnosis of inflammatory bow el disease. Peripheral arthritis, peripheral arthralgia
withoutjointswellingoreffusion,degenerativejointdiseaseorseropositivearthritiscan
occur in inflammatory bow el disease. In patients with peripheral arthralgia and
peripheral arthritis, there is a significantly greater prevalence of mucocutaneous
manifestationsofIBDi.e.oralulceration,erythemanodosum,pyodermagangrenosum,
anduveitis.

Dr.KhalidYusufElzohrySohagTeachingHospital2012
94
www.MRCPass.com

Rheumatology

RheumatologyQ090

A professional tennis player presents with shoulder pains especially whilst serving the
ball. He has limited passive and active shoulder abduction to less than 60. His
temperatureis36.5Candhehasanormalwhitecellcount.Thereistendernessaround
theanteriorportionoftheshoulderjoint.

Whichdiagnosisislikely?
A.Glenohumeraljointosteoarthritis
B.Bursitis
C.Tenniselbow
D.Supraspinatustendonitis
E.Septicarthritis

Answer:d)supraspinatustendonitis.

Pain during abduction with limitation of movement is suggestive of supraspinatous


tendonitis. Palpation or compression around the greater tubercle of the humerus is
particularlytender.

Dr.KhalidYusufElzohrySohagTeachingHospital2012
95
www.MRCPass.com

Rheumatology

RheumatologyQ091

A45yearwomanhasraynaudsphenomenon.Shealsohasdifficultyinswallowingand
dyspnoea.
Echocardiographyshowsrightheartstrain.Bloodtestsrevealrenalimpairment.

Whichoneofthefollowingantibodiesisspecifictothislady'scondition?
A.Anticentromereantibody
B.TopoisomeraseI
C.AntidsDNAantibody
D.AntiSCL70antibody
E.Antimitochondrialantibody

Answer:d)antiSCL70antibody.

AntiSCL70antibody(topoisomeraseI)istypicallyfoundinprogressivesystemicsclerosis
(notthelimitedcutaneousform,CREST).

Dr.KhalidYusufElzohrySohagTeachingHospital2012
96
www.MRCPass.com

Rheumatology

RheumatologyQ092

A55yearoldmanpresentswitha6weekhistoryoflethargyanddiffusepurpuricrash.
He is noted to have a right foot drop and a left ulnar nerve palsy. He complains of
arthralgiabuthasnoclinicalevidenceofinflammatoryjointdisease.Echocardiogramis
unremarkable, blood cultures are negative, ESR 80 mm/hr, ANCA negative, ANA
negative,rheumatoidfactorstronglypositive,C31.1g/l(0.751.6),C40.03g/l(0.14
0.5).
Dipstickurinalysisshowsblood++.

Whatisthelikelydiagnosis?
A.Takayasu'sarteritis
B.ANAnegativeSLE
C.Culturenegativeendocarditis
D.Cryoglobulinaemia
E.Rheumaticfever

Answer:d)Cryoglobulinaemia.

A low C4 together with a strongly positive rheumatoid factor suggests


cryoglobulinaemia as a cause of mononeuritis multiplex and rash. Palpable purpura,
arthralgia, hepatosplenomegaly, diffuse proliferative glomerulonephritis, Raynaud's
phenomenonandthrombosismayoccur.
Type I cryoglobulinaemia may be associated with lymphoproliferative disorders,
multiple myeloma, and monoclonal gammopathy of uncertain significance, and
macroglobulinaemia.Plasmapheresismayreducethelevelsofcryoglobulin.
Type II cryoglobulinaemia (mixed monoclonal) is usually composed of a monoclonal
component(usually IgG, IgMor IgA) and a polyclonal component(mainly IgG). Causes
are connective tissue diseases, Hepatitis B and C infection, infectious mononucleosis
andlymphoma.

Dr.KhalidYusufElzohrySohagTeachingHospital2012
97
www.MRCPass.com

Rheumatology

RheumatologyQ093

A 70 year old woman with a history of multiple myeloma has a serum calcium of 2.9
mmol/l.Heisprescribedpamidronateinfusionover4days.

Whatisitsmechanismofaction?
A.Promotescalcitonin
B.Increasescalcitriollevels
C.Inhibitosteoclasts
D.inhibitosteoblasts
E.Stimulateparathyroidhormonesecretion

Answer:C)inhibitosteoclasts.

Bisphosphonates inhibits osteoclasts and reduces progression tow ards bone


destruction.

Dr.KhalidYusufElzohrySohagTeachingHospital2012
98
www.MRCPass.com

Rheumatology

RheumatologyQ094

A 80 year old lady presents with a 5 day history of severe left temporal headache
radiating from her eye to the scalp. She had also experienced jaw discomfort during
eating.

Whichofthefollowingdrugsshouldbegivenwhileawaitingresultsofdiagnostictests?
A.Carbamazepine
B.Prednisolone
C.Azathioprine
D.Infliximab
E.Intravenousimmunoglobulin

Answer:b)Prednisolone.

The history suggests temporal arteritis. In view of the vision threatening nature of
disease,thepatientshouldbecommencedonsteroids.Typically60mgofprednisolone
perdayisrecommended.

Dr.KhalidYusufElzohrySohagTeachingHospital2012
99
www

w.MRCPass.ccom Rheum
matology

RheumatollogyQ0955

A30 0yearoldw
womanhassa3month hhistoryof arthralgia. Thereissw
wellingofth
hedistal
inteerphalangeaaljointsofttheringfingersofthehand.The wristonth herightanddankles
are swollen ass well. Onyycholysis wwas noted on the nails. Her serum inflam mmatory
marrkersareraised.

Whiichofthefoollowingistthelikelydia
agnosis?
A.Polym
myalgiarheumatica
B.Rheumatoidarthrritis
C.Gout
D.Psoriaaticarthropaathy
E.SLE

Answer:d)Pso
oriaticarthro
opathy.

Psoriaticarthritiseffectsd distalinterp phalangealjjointstendssbeasymm metrical.


Theereare5typ pesofpsoriaaticarthritis.
Asymmetrical oligoarticular arthritis is thoughtt to be the most comm mon type. Usually,
the digits of the
t hands and feet aare affected d first, with
h inflammaation of thee flexor
tenddon and synovium
s o
occurring ssimultaneou usly, leadin ng to the typical "saausage"
app
pearance(daactylitis).
Thee other typees are symmetrical po olyarthritis, DIP arthro
opathy, arth
hritis mutilaans and
spondylitiswithorwithou utsacroilitiss.


Psoriaticarthro
opathy

Dr.K
KhalidYusuffElzohrySo
ohagTeachin
ngHospital2012
100
www.MRCPass.com

Rheumatology

RheumatologyQ096

A60yearoldladyhassevererheumatoidarthritis.SheiscurrentlyonMethotrexate20
mg weekly for the past 5 months and also has been receiving regular infusions of
Infliximab. Her joint disease dramatically improved. She now presents with fevers,
coughandthereisevidencealargeleftsidedpleuraleffusiononherCXR.

Whatisthelikelydiagnosis?
A.Methotrexatepneumonitis
B.CMVinfection
C.Tuberculosis
D.Bronchialcarcinoma
E.Rheumatoidrelatedpulmonaryfibrosis

Answer:c)tuberculosis.

Serious opportunistic infections have been associated with the anti TNF alpha drug
infliximab, but the frequency of TB exceeds that associated with other infections.
Infliximabmayincreasetheriskoflymphoma.

Dr.KhalidYusufElzohrySohagTeachingHospital2012
101
www.MRCPass.com

Rheumatology

RheumatologyQ097

A75yearoldmanpresentswithanacuteonsetofseverepainandswellingoftheleft
elbow.Hementionsthathehadachestinfection3weeksago.Onexamination,hehad
atemperatureof38Candtheleftelbowwaserythematous,swollenandtender.

Whatisthemostappropriateinvestigation?
A.Creactiveprotein
B.Fullbloodcount
C.Jointaspiration
D.Uricacidlevel
E.Xrayofthejoint

Answer:C)jointaspiration.

This patient is likely to have reactive arthritis. How ever, gram stain and culture are
necessarytoexcludesepticarthritis.

Dr.KhalidYusufElzohrySohagTeachingHospital2012
102
www.MRCPass.com

Rheumatology

RheumatologyQ098

A75yearoldmanhassignificantbonypainswhichhavebeenoccuringfor3years.He
presents to the clinic for assessment and the investigations results were obtained
below:
Correctedcalcium2.4(2.22.6)
ESR20mm/1sthr
Alkalinephosphatase625iu/L(50100)
gammaGT42iu/L(1050)
Prostatespecificantigen7.4pg/L(06)

Whatisthemostlikelydiagnosis?
A.Osteoporosis
B.Osteomalacia
C.Metastaticprostaticcarcinoma
D.Paget'sdisease
E.Multiplemyeloma

Answer:d)Paget'sdisease.

Paget'sdiseasecausesahighalkalinephosphataseandnormalcalciumlevels.
Paget'sDiseaserepresentsanimbalanceofboneformationandresorption.Ittypically
beginswithexcessiveboneresorptionfollowedbyexcessiveboneformation.Themain
disturbanceisanexaggerationofosteoclasticboneresorption.
Themostcommonsitesofinvolvementincludethespine,pelvis,skull,femurandtibia.
Skullinvolvementmayproduceenlargementoftheheadcharacterizedbymoreevident
frontalbossinganddilatedsuperficialcranialmuscles.
Conductiveand/orsensorineuralhearinglossmayresultfromdiseaseofthetemporal
boneorossicles.

Dr.KhalidYusufElzohrySohagTeachingHospital2012
103
www.MRCPass.com

Rheumatology

RheumatologyQ099

A75yearoldpatientwithosteoarthritisofthekneehasbeentakingcodeine30mgqds
andalsoparacetamolregularly.Hecontinuestohavekneepains.

Whatisthenextbesttreatment?
A.Diclofenac
B.Morphine
C.Ibuprofengel
D.Higherdoseofcodeine
E.Oralhydrocortisone

Answer:C)ibuprofengel.

NSAIDS tend to be better for pain control in osteoarthritis, but have significant side
effects. Local NSAID application should be considered as well as intraarticular steroid
injections.

Dr.KhalidYusufElzohrySohagTeachingHospital2012
104
www

w.MRCPass.ccom Rheum
matology

RheumatollogyQ1000

A 40 year old lady presents with a swollen rigght knee. This was asp
pirated. Un
nder the
polaarised microscope, theere were neutrophils +++ and so ome blue rhhomboidal crystals
undderparallelpolarisedligght.

Themostlikelyydiagnosisiis:
A.Osteo
oarthritis
B.Rheumatoidarthrritis
C.Gout
D.Pseud
dogout
E.Septiccarthritis

Answer:d)Pseeudogout.

Thee blue colo


our and po
ositively birefringent crystals un
nder polarised light suggest
pseudogout.Neutrophilleevelsarehigghininfectiion,goutan
ndpseudogout.

Dr.K
KhalidYusuffElzohrySo
ohagTeachin
ngHospital2012
105
www.MRCPass.com

Rheumatology

RheumatologyQ101

A45yearoldmanhasarenaltransplant.Hepresentswithahotswollenankleonthe
leftandfevers.Thesymptomshavebeenpresentfor2days.

Whatshouldbedone?
A.Checkserumuricacidlevel
B.Injectintraarticularsteroids
C.Givecolchicine
D.Sendjointfluidformicroscopyandculture
E.Intravenouscefuroxime

Answer:d)Sendjointfluidformicroscopyandculture.

The differentialdiagnosis isgout(commoninrenalpatients)butthe patientwillbein


immunosupressionandsepticarthritisshouldbeexcludedwithjointfluidculture.

Dr.KhalidYusufElzohrySohagTeachingHospital2012
106
www

w.MRCPass.ccom Rheum
matology

RheumatollogyQ1022

A40
0yearoldaathletehasp
painonabdductionofh
herarm,parrticularlywhenresiste
ed.

Whiichtendonp pathologyiisaffected?
A.Bicep
pstendonitis
B.Supraaspinatusten
ndonitis
C.Teressminortendo
onitis
D.Infrasspinatustend
dinitis
E.Subsccapularistendonitis

Answer:b)Sup
praspinatustendonitis.

Pain
ninabductionupto90
0degreesissduetosup
praspinatustendonitis.


Illusstrationofssupraspinatusmuscle

Dr.K
KhalidYusuffElzohrySo
ohagTeachin
ngHospital2012
107
www.MRCPass.com

Rheumatology

RheumatologyQ103

A 60 year old man has worsening discomfort in both shoulders. He is haemodialysis


dependent. Past medical history included bilateral carpal tunnel decompression. His
Investigations reveal: haemoglobin 9.8 g/dl, ESR 35 mm/1st hr, C reactive protein 15
mg/L,Urate0.58.

Whatisthelikelydiagnosis?
A.Reiter'ssyndrome
B.Amyloidosis
C.Polymyalgiarheumatica
D.Gout
E.Osteomalacia

Answer:b)amyloidosis.

b2microglobulindepositioninjointsmayleadtoamyloidosis. Thiscanoccur10 years


onfromdialysis,andleadtocarpaltunnelsyndrome.

Dr.KhalidYusufElzohrySohagTeachingHospital2012
108
www.MRCPass.com

Rheumatology

RheumatologyQ104

A45yearoldmanpresentswithapainfulswollenknee.Hefeelsgenerallyunwelland
has fever. He has recently had a flulike illness, an erythematous rash on the trunk
followedbyaselflimitingepisodeofdiarrhoea.Adiagnosisofreactivearthritisismade
bytherheumatologist.

Howshouldthepatientbemanaged?
A.Highdosesteroids
B.Broadspectrumivantibiotics
C.Arthroscopyandwashoutofthejoint
D.Ifthesymptomsbecomeschronic,sulphasalazinemaybeuseful
E.Bonescantolookforafocusofinfection

Answer:d)ifthesymptomsbecomeschronic,sulphasalazinemaybeuseful.

The diagnosis is likely to be reactive arthritis following an infectious illness. Although


jointaspirationmaybeuseful,thereisnoneedforarthroscopyatpresent.NSAIDsmay
be used for symptoms control now. If the symptoms persist, sulphasalazine or
methotrexatemaybeuseful.

Dr.KhalidYusufElzohrySohagTeachingHospital2012
109
www

w.MRCPass.ccom Rheum
matology

RheumatollogyQ1055

A166girlisinveestigatedfo
orswellingaandpainintherightw
wrist,leftkneeandrigh
htankle.
InveestigationssshowapositiveANA1:160withveRheumaatoidfactor..

Wha atissheatriskofdeveeloping?
A.Psoriaasis
B.Butteerflyfacialrassh
C.Erosivvejointdiseaase
D.Uveittis
E.Bamb
boospine

Answer:D)uve
eitis.

Juveenilechroniicarthritisisatermusedtodescrribearthritissoccurringinsomeoneewhois
lesss than 16 years old that lasts forr more than
n three mo
onths. Largee joints tend to be
affeected.Rheumatoidfacttorisoften negative,aandthereispositivean ntinuclearantibody
especially
e in pauciartticular JCA.. Acute anterior uveeitis is most commonly in
pauciarticular juvenile ch hronic arthritis. Stiffneess, amyloiidosis, and osteoporo osis also
occu ur.


Uveeitiscausinggredeye(leenticularpreecipitatesm
maybeseen
n)

Dr.K
KhalidYusuffElzohrySo
ohagTeachin
ngHospital2012
110
www

w.MRCPass.ccom Rheum
matology

RheumatollogyQ1066

A22
2yearoldmmanpresentswitha4w weekhistorryofapainffulswollen leftknee.H
Hehasa
pastt medical history of a treated sexually transmitted disease 6
6 months ago.
a On
examinationth herewasalargeeffusio
oninthelefftknee.
Synovialfluidaanalysissho
owsawhite cellcounto of15x10^9
9/lbutcultu
urewasneggative.

Whiichoneofth hefollowing
gorganismsisthemosstlikelycau
use?
A.Gardn
nerella
B.Chlam
mydia
C.Trepo
onemapallidum
D.Neissseriagonorrh
hoea
E.Tricho
omonasvaginalis

Answer:d)Neisseriagono
orrhoea.

Gonnococcal arrthritis is caaused by aan infection


n with the gramnegattive diplocooccus N
gonnorrhoeae. Neisseria gonorrhoea
g a occurs in
n young adults, ofteen preceded by a
miggratoryarthritis.
Thee bacteremiic phase is a classic ttriad of migratory polyarthritis, tenosynovitis, and
dermmatitis. This patient was
w treated previouslyy for a sexu ually acquire
ed infection
n hence
mayybethereaasonfortheeculturetobenegativee.
Thee initial treatment of choice forr gonococccal arthritiss is a third
dgeneration beta
lactamaseresiistant ceph halosporin (eg, ceftriiaxone, cefftizoxime, cefotaximee) or a
pennicillin,iftheeorganismissensitive.


Mulltipleintraccellulargram
mnegatived
diplococcic

Dr.K
KhalidYusuffElzohrySo
ohagTeachin
ngHospital2012
111
www.MRCPass.com

Rheumatology

RheumatologyQ107

A35yearoldladyhas positiveANA,andhasabutterfly shaped rashonherface.Her


physician makes a diagnosis of SLE. She has flare ups of joint swellings and pains
requiring several months treatment with prednisolone. 1 year later she presents with
hippainlimitinghermobility.

Whichoneofthefollowingisalikelycause?
A.Rheumatoidarthritis
B.Septicarthritis
C.Juvenilechronicarthritis
D.Avascularnecrosis
E.Perthe'sdisease

Answer:d)avascularnecrosis.

15% of patients with SLE develop avascular necrosis of the bone. Nephritis, vasculitis
andlongtermsteroidusepredisposetoavascularnecrosis.

Dr.KhalidYusufElzohrySohagTeachingHospital2012
112
www.MRCPass.com

Rheumatology

RheumatologyQ108

A50yearoldmanpresentswithchronicrecurrentsinusitisandoccasionalhaemoptysis.
HehasmiddlelobepatchyshadowingonhischestXray.Hiscreatinineis145andurine
dipstickshows++proteinandblood.

Whichtestresultismostlikely?
A.PositivecANCA
B.PositiveANA
C.PositiveKveimtest
D.PositiveASOT
E.SputumAFBpositive

Answer:a)positivecANCA.

Nasalcavity,pulmonaryandrenalgranulomatousinvolvementisclassicalinWegener's
granulomatosis. 70% of patients have a positive cANCA. In addition, w hen ANCA is
positive, PR3 (Wegener's) and MPO (microscopic polyangitis) helps to distinguish
Betweenthetwodifferentialdiagnoses.

Dr.KhalidYusufElzohrySohagTeachingHospital2012
113
www.MRCPass.com

Rheumatology

RheumatologyQ109

A55yearoldfemalepresentscomplainingofbonepainsandmuscularweakness.Her
gaitiswaddlinginnature.
Xrayshowspseudofracturesofherpubicrami.
Investigations show : Serum corrected calcium 2.05 mmol/L, phosphate 0.43 mmol/L,
alkalinephosphatase230U/L.

Whatisthediagnosis?
A.Pseudohypoparathyroidism
B.Secondaryhyperparathyroidism
C.Osteopetrosis
D.Osteomalacia
E.Paget'sdisease

Answer:d)osteomalacia.

The clinical radiological and biochemical features in this patient suggest she has
osteomalacia. Osteomalacia is characterized by a low serum calcium and phosphate
withelevatedserumalkalinephosphatase.
Osteomalacia may be caused by deficiency of vitamin D or phosphate deficiency.
Malabsorption syndromes, renal failure and liver disease can result in vitamin D
deficiency.

Dr.KhalidYusufElzohrySohagTeachingHospital2012
114
www

w.MRCPass.ccom Rheum
matology

RheumatollogyQ1100
A22yearold Armenianm manpresen ntswithpainandswellingoftheleftknee. Healso
givees a historry of recurrrent episo
odes of fevver, pleuritic pains in the cheest, and
generalised raash. These episodes typically laast for 23 3 days. Exxamination reveals
spleenomegaly,swollenkneesandankkles.Urined dipstickrevveals2+prooteinuria.
Wha atisthelikeelydiagnosiis?
A.Ankyllosingspond
dylitis
B.Reacttivearthritis
C.FamilialMediterraaneanfever
D.Tuberculosis
E.Dermatomyositis

Answer:c)Fam
milialMediteerraneanfeever.
Fammilial Mediterranean fever is an inherited d condition n characterrized by reecurrent
epissodesofpaainful inflam
mmation in the abdom men, chest, orjoints. These
T episoodesare
ofteenaccompaaniedbyfevverandsom metimesa rash.Thefirstepisodeeusuallyocccursby
the age of 20 years, butt in some cases, the initial attack occurs much
m laterr in life.
Typically,episo
odeslast12 2to72hourrsandcanvvaryinseveerityandin thelength oftime
Betwweenattacks.
AA amyloidosis commonlly involves the kidneyys, spleen aand GI tract. Colchicin ne given
prophylacticallyinFMFoffferssome protection againsttheedevelopm mentofamyyloidosis
inm
mostpatientts.
Fammilial Mediterranean fever prim marily affeects populaations origginating fro om the
Med diterraneann region, particularly
p people of Armenian,, Arabic, Turkish,
T andd North
African Jew ish ancestry.. Mutationss in the MEFV gene cause
c familial Mediterranean
feveer.


Rashseeninfaamilialmediiterraneanffever

Dr.K
KhalidYusuffElzohrySo
ohagTeachin
ngHospital2012
115
www

w.MRCPass.ccom Rheum
matology

RheumatollogyQ1111

A 35 year old man presents acutelyy with ureth


hritis, conju
unctivitis an
nd arthritis. He has
beeenhavingjointpainsinthewristandhips.
Oneexamination,hehasarashontheesolesofhisfeetandaalsocircinattebalanitis..
Rheeumatoidfactorisnegaativeandheehasraised dinflammattorymarkerrs.

Wha atisthedia
agnosis?
A.Behceet'ssyndrom
me
B.Rheumatoidarthrritis
C.Reiter'ssyndromee
D.Psoriaaticarthropaathy
E.Ankylosingspondylitis

Answer:c)Reitter'ssyndro
ome.

Reitterssyndro
omeisurethritis,conju unctivitis,se
eronegativeearthritis(ccannotsee,,cannot
peee, cannotclimb atree)).Thetypiccalpatientiisayoung manwithrrecentureth hritis or
dyseentery.Theeseronegatiivearthritissisusuallyaamonooro
oligoarthritiss.
Other featuress are anterior uveitis, keratoderm ma blenorrhagica (broow n absceesses on
palmmsandsolees),mouth ulcers,plan ntarfasciitissandarchilllestendinittis(entheso
opathy),
circinatebalanitis(painlesssrash)anddaorticinco ompetence.Managem mentisusuaallywith
resttandNSAIDDs.


KeraatodermaB
BlenorrhagiccainReiter'ssyndromee

Dr.K
KhalidYusuffElzohrySo
ohagTeachin
ngHospital2012
116
www.MRCPass.com

Rheumatology

RheumatologyQ112

A 50 year old patient with rheumatoid arthritis has the following full blood count
results:Haemoglobin10.5g/dL,Platelets450x10^9/L,WhiteCellCount8.5x10^9/L,
MCV103fL.

Whichdrugisthelikelycauseofthis?
A.Infliximab
B.Ciclosporin
C.Leflunomide
D.Prednisolone
E.Methotrexate

Answer:e)Methotrexate.

MethotrexatemayleadtomacrocytosisasaresultofB12orfolatedeficiency.It may
also be associated with bone marrow suppression, causing leucopenia or
thombocytopaenia. Methotrexate may also cause mouth ulcers, stomatitis, cough and
dyspnoea.

Dr.KhalidYusufElzohrySohagTeachingHospital2012
117
www.MRCPass.com

Rheumatology

RheumatologyQ113

A32yearoldmanhashadayearshistoryofbilateralhippainsandbackpains.Thereis
notpastmedicalhistoryoftraumatotheback.Nonsteroidalantiinflammatorydrugs
helpedtorelievehissymptoms.

Whatisthelikelydiagnosis?
A.Gluteusmediustendonitis
B.Osteoarthritis
C.Hipfracture
D.Sacroilitis
E.Osteoarthrosis

Answer:D)sacroilitis.

Painandstiffnessinthelowerbackorbuttocks,especiallyinthemorningistypicalof
sacroilitis. It is typically helped by NSAIDS or steroids. X rays will help to confirm the
diagnosis.Itisassociatedwithvariousinflammatorydiseasese.g.ankylosingspondylitis,
psoriaticarthritis.

Dr.KhalidYusufElzohrySohagTeachingHospital2012
118
www.MRCPass.com

Rheumatology

RheumatologyQ114

A35yearoldladypresentswithstiffness,painandswellingofherhandsandwrists.On
examinationshehasfirmsubcutaneousnodulesoverherelbows,swellingofherwrists,
ulnardeviationatthemetacarpophalangealjoints,dinnerforkdeformityandswellingof
herproximalinterphalangealjoints.
Investigationsrevealanormocytic,normochromicanaemia,elevatedESRandCRP.

Which is the most commonly used test which could determine whether she is sero
positive?
A.IgAantibody
B.IgMantibody
C.IgEantibody
D.IgDantibody
E.IgGantibody

Answer:b)IgMantibody.

Thepatienthasfeaturesofseropositiverheumatoidarthritis(presenceofrheumatoid
factor).RheumatoidfactorisacirculatingantibodydirectedagainsttheFcfragmentof
immunoglobulin. The antibody may IgM, IgG, or IgA. The commonly employed test
detectstheIgMrheumatoidfactor.

Dr.KhalidYusufElzohrySohagTeachingHospital2012
119
www.MRCPass.com

Rheumatology

RheumatologyQ115

A32yearoldmanhasa6monthhistoryofdryeyesandmouth.Onexamination,there
wasevidenceofkeratoconjunctivitis,parotidglandenlargementandaSchirmer'stestis
positive.
HisbloodtestsrevealapositiveANA,RoandLaextranuclearantigensarealsopositive.

Whatisthediagnosis?
A.SLE
B.Rheumatoidarthritis
C.Sjogren'ssyndrome
D.Wegener'sgranulomatosis
E.Pulmonaryeosinophilia

Answer:c)Sjogren'ssyndrome.

The patient has Sjogren's syndrome. Sjgren syndrome is a chronic autoimmune


disorder characterized by xerostomia (dry mouth), xerophthalmia (dry eyes), and
lymphocytic infiltration of the exocrine glands. This triad is also known as the sicca
complex.
The Schirmer test is probably the only test available in the ED to strongly support or
refutesuspicionofSjgrensyndrome.Ateststripoffilterpaperisplacednearthelower
conjunctivalsactomeasuretearformation.Apositivetestoccurswhenlessthan5mm
offilterpaperiswetafter5minutes.
Rheumatoidfactor,ANA,RoandLaantigensarecommonlypresent.
As a result of the lymphocytic infiltration, 10% of patients may develop
pseudolymphoma,alymphoproliferativeprocess.Approximately10%ofthesepatients
candevelopnonHodgkinlymphoma(1%ofallpatientswithSjgrensyndrome).

Dr.KhalidYusufElzohrySohagTeachingHospital2012
120
www.MRCPass.com

Rheumatology

RheumatologyQ116

A38yearoldladypresentswithmyalgiaandlethargy.Herbloodtestsshowapositive
ANAwithatitreof1:1024andrheumatoidfactorisnegative.
TheCKisraisedat360U/l.ExtranuclearantigentestsshowanegativeRoandnegative
La,negativeScl70andpositiveribonuclearproteinantibodyat160units.

Whatisthediagnosis?
A.Polymyalgiarheumatica
B.Polymyositis
C.Scleroderma
D.Systemiclupuserythematosus
E.Mixedconnectivetissuedisease

Answer:e)mixedconnectivetissuedisease.

A positive ANA (speckled pattern), raised CK and positive anti RNP antibody suggests
mixedconnectivetissuedisease.

Dr.KhalidYusufElzohrySohagTeachingHospital2012
121
www.MRCPass.com

Rheumatology

RheumatologyQ117

A 50 year old man presents to the renal team with uraemic symptoms. He also has
markedlyreducedrangeofmovementatthespinewithaSchoberstestof10mm.
HisCRPis102mg/dlandESR98mm/hr.Urinedipstickshowsproteinuria++++.
He has had treatment with penicillamine in the past for arthritis. He takes regular
ibuprofen.

Whatisthemostlikelycauseofhisrenalsymptoms?
A.Nephriticsyndrome
B.NSAIDnephropathy
C.Scleroderma
D.ALamyloid
E.AAamyloid

Answer:e)AAamyloid.

Nephrotic range proteinuria and renal failure in the context of a prolonged untreated
inflammatoryresponsesuggestsAAamyloid.SincecirculatingserumAAistheprecursor
ofAAamyloiddeposits,reductionoftheprecursorproteinisthemostrationalapproach
atpresentforthemanagementofamyloidosis.
Prevention of amyloidosis is preferable to treatment of the established disease. Thus
aggressivetherapyofrheumaticdiseasessuchasRAandJCAisdesirableinthiscontext.

Dr.KhalidYusufElzohrySohagTeachingHospital2012
122
www.MRCPass.com

Rheumatology

RheumatologyQ118

A 45 year old woman presents with confusion. On examination she was pyrexial, had
livedoreticularishadabloodpressureof190/100mmHg.
Examinationoftheabdomenrevealedleftflanktenderness.
Investigationsrevealed:
Hb13.9g/dL
whitecellcount6.5x10^9/L
plateletcount110x10^9/L
serumcreatinine95umol/L
urinedipstickanalysis:blood+++,protein+

Whichoneofthefollowingtestsislikelytobepositive?
A.AntiRoantibody
B.AntiGBMantibody
C.Anticardiolipinantibody
D.ASOT
E.ANCA

Answer:c)Anticardiolipinantibody.

The diagnosis is SLE and antiphopholipid syndrome. The presentation would be


consistent with renal vein thrombosis (flank pain and proteinuria). Antiphospholipid
syndrome(APS)isadisordercharacterizedbyrecurrentvenousorarterialthrombosis
and/or fetal losses associated with typical laboratory abnormalities. These include
persistently elevated levels of antibodies directed against membrane anionic
phospholipids(ie,anticardiolipin[aCL]antibody,antiphosphatidylserine).
Vascular thrombosis DVT, MI, CVA or miscarriages may occur. Other features are
nonthrombotic neurologic symptoms, such as migraine headaches, chorea, seizures,
transverse myelitis, GuillainBarr syndrome, thrombocytopenia or hemolytic anemia,
Livedoreticularis,AvascularnecrosisofboneandPulmonaryhypertension.
Aspirinorwarfarinisrecommendedforpatientswiththromboticsyndromes.

Dr.KhalidYusufElzohrySohagTeachingHospital2012
123
www.MRCPass.com

Rheumatology

RheumatologyQ119

A60yearoldladyhaspolyarthropathyduetoRheumatoidArthtitis.

Whichoneofthefollowingmoleculesplaysacentralroleinitspathogenesis?
A.IFNgamma
B.Interleukin8
C.TNFalpha
D.Endotoxin
E.Nitricoxide

Answer:C)TNFalpha.

Inthecontextofrheumatoidarthritis,TNFhasinvolvementincytokineregulation,cell
recruitment,angiogenesis,andtissuedestruction.HenceantiTNFalphaantibodiessuch
asinfliximabareusedintherapy.

Dr.KhalidYusufElzohrySohagTeachingHospital2012
124
www.MRCPass.com

Rheumatology

RheumatologyQ120

A 50 year old woman presents with breathlessness. She has a 2 year history of
Raynaud's.Onexamination,shehadtelangiectasiaandtightskinaroundthemouth.Her
investigationsshowanESRof70mm/hrandpositiveanticentromereantibodies.

Whichofthefollowingisatypicallatecomplicationofthisdisorder?
A.Renalhypertensivecrisis
B.Lungmalignancy
C.Pulmonaryhypertension
D.Myositis
E.Mitralregurgitation

Answer:c)Pulmonaryhypertension.

ThediagnosisisCRESTsyndrome.Renalhypertensivecrisisismorecommonindiffuse
systemic sclerosis and pulmonary hypertension is more common in limited cutaneous
scleroderma.

Dr.KhalidYusufElzohrySohagTeachingHospital2012
125
www.MRCPass.com

Rheumatology

RheumatologyQ121

An 18 year old girl developed pulmonary haemorrhage and subsequent acute renal
failurerequiringdialysis.Arenalbiopsyshowscrescenticglomerulonephritis.

Whichoneofthefollowingantibodiesislikelytobepresent?
A.Antimitochondrial
B.Anticentromere
C.Antinuclear
D.Antiphospholipid
E.Antimyeloperoxidase

Answer:e)Antimyeloperoxidase.

ThispatientmanifestsapulmonaryrenalsyndromewhichiscommonlyduetoanANCA
positive vasculitis. P ANCA which correlates with antimyeloperoxidase (MPO)
antibodies, is highly sensitive and specific tow ards rapidly progressive
glomerulonephritisandhaemorrhagicalveolarcapillaritis.Lesscommonlythiscouldbe
duetoGoodpasturessyndrome(antiGBMantibodies).

Dr.KhalidYusufElzohrySohagTeachingHospital2012
126
www

w.MRCPass.ccom Rheum
matology

RheumatollogyQ1222

A 65 year old man with chronic leu


ukaemia presents withh symptom
ms of gout. He was
giveenAllopurinnol.

HowwdoesAllop purinolprevventaccumu ulationofu


uricacid?
A.Byinccreasinguriccacidmetabo
olism
B.Byen
nhancingitsssolubility
C.Byinh
hibitingpurin
nesynthesis
D.Byinh
hibitingpyrim
midinesynth
hesis
E.Byinh
hibitingxanthineoxidasee

Answer:E)Byiinhibitingxaanthineoxid
dase.

Allo
opurinol inh
hibits xanth
hine oxidasse, the enzzyme that catalyzes the converrsion of
hypoxanthinettoxanthineeandofxan nthinetouuricacid.Hyypoxanthineeandxanthhineare
breaakdownpro oductsofpuurine.

Dr.K
KhalidYusuffElzohrySo
ohagTeachin
ngHospital2012
127
www.MRCPass.com

Rheumatology

RheumatologyQ123

A 32 year old woman in the third trimester of her second pregnancy develops acute
onsetrightgroinpain.
Onexamination,allrighthipmovementsarepainfulandsheistenderintherightgroin
and over the greater trochanter. Lumbar spine examination shows an exaggerated
lordosisonlyandtherearenoneurologicallowerlimbdeficits.Initialplainfilmsofthe
hiparenormal.

Themostlikelydiagnosisis:
A.Septicarthritis
B.Hipdislocation
C.Avascularnecrosisofthehip
D.Pregnancyexacerbatingosteoarthritis
E.Pseudogout

Answer:c)Avascularnecrosisofthehip.

Thisisaclassicalpresentationofavascularnecrosisofthefemoralheadinpregnancy.
Transientosteoporosisofthehipisalsoarecognisedcauseofhippaininpregnancy,but
plainradiographswouldusuallyshowmarkedunilateralosteopeniaofthefemoralhead
andacetabulum.
Thearterialsupplytothefemoralheadiseasilydamagedwithanyfemoralneckfracture
displacement.
Atraumatic causes of this are : Alcohol abuse, Chemotherapy, Chronic liver disease,
Corticosteroids, Gaucher disease, Gout, Hemoglobinopathy (eg, sickle cell disease),
Metabolic bone disease, Pregnancy, Radiation, Systemic lupus erythematosus,
Vasculitis.

Dr.KhalidYusufElzohrySohagTeachingHospital2012
128
www.MRCPass.com

Rheumatology

RheumatologyQ124

A55yearoldwomanwithpsoriasishassignificantjointpains.

Whichoneofthefollowingiseffectiveinthetreatmentofpsoriaticarthropathy?
A.Codeinephosphate
B.Methotrexate
C.Betainterferon
D.Capacitabine
E.Buprenorphine

Answer:b)Methotrexate.

NSAIDs,sulphasalazine,methotrexateandTNFalphaantagonistsareusefulinpsoriatic
arthropathy.

Dr.KhalidYusufElzohrySohagTeachingHospital2012
129
www.MRCPass.com

Rheumatology

RheumatologyQ125

A 65 year old woman attends has a 12 week history of lethargy, neck pains and
weaknessinthelowerlimbs.
She has a long history of lower back pains and generalised osteoarthritis. She takes
diclofenacregularly.
Onexaminationthereiswastingofherupperlimbs.Tonemildlyincreasedinthelower
limbs.Thereisinversionofrightsupinatorreflex,triceps,kneeandanklejerksarebrisk
bilaterally.Rightplantarisextensorandtheleftisflexor.
Investigationsshow:
Hb11.6g/l
WCC8x10^9/l
Plat160x10^9/l
ESR73mm
CRP12mg/l
Na138mmol/l
K4.4mmol/l
Urea5.8mmol/l
Creatinine95umol/l
Protein83g/l
Albumin32g/l
Ca2.33mmol/l
Xraycervicalspineshowsextensiveosteophytes

Whatisthelikelydiagnosis?
A.Ankylosingspondylitis
B.Polymyalgiarheumatica
C.Cervicalspondyloarthropathy
D.Multiplesclerosis
E.Syringomyelia

Answer:c)Cervicalspondyloarthropathy.

Cervicalspondylosismaypresentwithassociatedpainsintheneckradiatingdownthe
armsandback.Theremaybeuppermotorneuronsignsintheupperandlowerlimbs.

Dr.KhalidYusufElzohrySohagTeachingHospital2012
130
www.MRCPass.com

Rheumatology

RheumatologyQ126

A 70 year man presents with right foot drop, hand numbness, fevers, malaise, weight
loss,polymyalgiaanddiffusejointpainsfor2months.
Onexamination,heappearsunwellandhasatemperatureof38C.
Investigationsreveal:
Hb8.5g/dL
erythrocytesedimentationrate95mm/hr
serumcreatinine220mol/L
urineanalysis:blood++
urinemicroscopy:whitecells&redcellcastsseen

Whichoneofthefollowingisthelikelydiagnosis?
A.Multiplemyeloma
B.Antiphospholipidsyndrome
C.Takayasu'sarteritis
D.Polyarteritisnodosa
E.Goodpasture'ssyndrome

Answer:d)polyarteritisnodosa.

This patient has a mononeuritis multiplex, fever and nephritic renal involvement
suggesting a diagnosis of polyarteritis nodosa. PAN causes transmural necrotizing
inflammationofsmallsizedormediumsizedmusculararteries.PANisararecondition.
Although the causes are unknown in most cases, there is an association with: Hep B
virus, Hep C virus, HIV, Cytomegalovirus, Parvovirus B19 and Human Tlymphotrophic
virus.
Approximately 20% of patients with classic PAN are positive for PANCA. Steroids
(prednisolone) and immunosuppressive (cyclophosphamide) medications form the
backbone of therapy. Plasma exchange is useful as a secondline treatment in PAN
refractorytoconventionaltherapy.

Dr.KhalidYusufElzohrySohagTeachingHospital2012
131

Você também pode gostar